75
KCONFLICT OF LAWS 2017 SOME GENERAL CONSIDERATIONS Keep in mind that while activities being dealt with may be transnational in nature, the law is not. The law is local. So, based on the conflicts rules applicable in different jurisdictions, you should ask: 1. Where do you want to sue? 2. What law will that court apply? 3. If I get a judgement, how will I get that judgement enforced? (i.e. will a jurisdiction where the D has assets enforce it?) Considerations to account for: 1. Characterization (occurs in jurisdiction, exclusionary rules, and D&R) 2. Jurisdiction 3. Exclusionary Rules 4. Renvoi 5. Incidental consideration Conflicts Rules Address three areas: 1. Jurisdiction: Can this court even hear the action? 2. Choice of Law: Provided there is jurisdiction, what law will be applied by the court? 3. Recognition & Enforcement (R&E): Will a jurisdiction R&E extra-territorial judgements? Definitions: Lex causae: the law chosen and applied by the forum court. PP: doctrine of comity and deference to other jurisdictions; justice and fairness for the litigating parties Lex fori: laws of the forum and are applied to the case. In personam: directed at a particular person; law suit against a specific person; goal: personal liabilities In rem: power about or against ‘the thing’; power that a court may exercise over property or a ‘status’ against a person, primary objective of action A. CHARACTERIZATION: SUBSTANCE & PROCEDURE When you characterize the cause of action, you are essentially determining what type of problem you are dealing with. It is a question of classification. Current test in Canada for determining w/ S or P: Which rules make the case run smoothly? (procedural) vs. which rules determine rights of both parties? (substantive) (Tolofson, SCC 1994) o Only justification for characterizing forum rule as procedural is the convenience of the court! Ask: What is being characterized; a. Categories of law, causes of action, and legal rules are all characterized in different situations. i. Example of characterization: Is this an issue of K or Tort? If latter, what is my cause(s) of action? Negligence? Invasion of privacy? Etc. Having 1

cans.allardlss.comcans.allardlss.com/.../cans/Edinger_76_Fall_2017_Jasmin…  · Web viewkConflict of Laws 2017. Some General Considerations. Keep in mind that while activities being

  • Upload
    others

  • View
    1

  • Download
    0

Embed Size (px)

Citation preview

Page 1: cans.allardlss.comcans.allardlss.com/.../cans/Edinger_76_Fall_2017_Jasmin…  · Web viewkConflict of Laws 2017. Some General Considerations. Keep in mind that while activities being

KCONFLICT OF LAWS 2017

SOME GENERAL CONSIDERATIONS

Keep in mind that while activities being dealt with may be transnational in nature, the law is not. The law is local. So, based on the conflicts rules applicable in different jurisdictions, you should ask:

1. Where do you want to sue? 2. What law will that court apply? 3. If I get a judgement, how will I get that judgement enforced? (i.e. will a jurisdiction where the D has assets

enforce it?)Considerations to account for:

1. Characterization (occurs in jurisdiction, exclusionary rules, and D&R)2. Jurisdiction3. Exclusionary Rules4. Renvoi5. Incidental consideration

Conflicts Rules Address three areas:

1. Jurisdiction: Can this court even hear the action? 2. Choice of Law: Provided there is jurisdiction, what law will be applied by the court? 3. Recognition & Enforcement (R&E): Will a jurisdiction R&E extra-territorial judgements?

Definitions: Lex causae: the law chosen and applied by the forum court. PP: doctrine of comity and deference to other

jurisdictions; justice and fairness for the litigating parties Lex fori: laws of the forum and are applied to the case. In personam: directed at a particular person; law suit against a specific person; goal: personal liabilities In rem: power about or against ‘the thing’; power that a court may exercise over property or a ‘status’ against

a person, primary objective of action

A. CHARACTERIZATION: SUBSTANCE & PROCEDURE

When you characterize the cause of action, you are essentially determining what type of problem you are dealing with. It is a question of classification. Current test in Canada for determining w/ S or P: Which rules make the case run smoothly? (procedural) vs. which rules determine rights of both parties? (substantive) (Tolofson, SCC 1994)

o Only justification for characterizing forum rule as procedural is the convenience of the court!

Ask: What is being characterized;a. Categories of law, causes of action, and legal rules are all characterized in different situations.

i. Example of characterization: Is this an issue of K or Tort? If latter, what is my cause(s) of action? Negligence? Invasion of privacy? Etc. Having characterized the category of law and causes of action at play—what is the lex causa? what law will be applied? If it’s the law of the forum, we’re done. But if it’s a foreign system of law, we then need to determine which laws will apply by determining what is substantive in nature vs. what is procedural.

Important b/c forum will always apply its own rules of procedure, even if different substantive law applied. So, if forum uses law other than its own to resolve dispute, will nonetheless have to determine which potentially applicable rules from each legal system are substantive vs. procedural.

Substance vs. procedure in characterization:

1

Page 2: cans.allardlss.comcans.allardlss.com/.../cans/Edinger_76_Fall_2017_Jasmin…  · Web viewkConflict of Laws 2017. Some General Considerations. Keep in mind that while activities being

Rule: Courts apply their own procedural rules (b/c administrative convenience, would be too much delay and expense if courts expected to apply foreign procedure).

Therefore, law must be characterized as substantive or procedural when forum’s choice of law rule has selected foreign lex causa (Lex Causa=law which forum selects to govern the merits of the case).

o If BC COL rule selects BC law (or counsel doesn’t argue otherwise) then BC law will govern whole of the action and classification of laws as to procedural or substantive doesn’t matter.

Forum Rule Characterization Lex Causae Rule Characterization Rule that Applies

Procedural Procedural Forum Rule

Substantive Substantive Lex Causae Rule

Procedural Substantive Both Rules Apply

Substantive Procedural Neither Rule Applies

You really want to avoid the last two scenarios (both or neither apply) so figure out how to characterize as first or second situation. If you end up with 3rd and 4th, go back until you can make it fit into 1 or 2. Generally, forum will use its own definitions, not those of lex causa, for purpose of internal consistency (e.g. “Penal”)

LIMITATIONS

TOLOFSON V. JENSEN (1994) SCC

F: MVA in Saskatchewan. P 12 y/old passenger in Dad’s car, sustains injuries. They are both residents of BC. Other driver was resident of Sask. When P reaches age of majority, brings PI action in BC. P wants BC law to apply b/c limitations and only has to establish negligence on behalf of Ds. Ds want Sask law applied b/c P is statute barred from brining action in Sask (1-yr limitation) and would have to establish gross negligence if proceeded to trial. I: What law governs the merits of this case, BC or Sask? If Sask, is limitation rule one of substance or procedure (if procedural, BC limitation rule applies, if substantive, Sask limitation rule applies). L: Law of place of tort (lex loci delicti) governs choice of law in tort action. SK limitation law was substantive, so applied to bar action. A: La Forest J. cites BCCA in Block Bros Realty: “legislation should be categorized as procedural only if question is beyond any doubt. If there’s any doubt … substantive”. SCC agrees while acknowledging it’s a departure from the traditional approach. Courts should characterize rules as procedural only where necessary for smooth operation of court machinery (in other words, where foreign lex loci selected under forums COL rules, there is a presumption against characterizing a rule of the forum as procedural , aka bias towards applying as much of chosen foreign law as possible when characterizing as S or P). R: Lex loci delicti is SK, so substantive SK law applies to the action, since limitation periods are substantive (confer rights on Ds) Sask limitation period applies.

STATUS TO SUE

Law in Canada: status to sue generally procedural, determined by the lex fori (Hamza, ABCA 1995)o Qualification – forum’s choice of law rules may determine status where it is otherwise lacking

Test for foreign entity without status under lex fori : (Hamza, ABCA 1995)o Foreign party must demonstrate:

1. There is an identifiable legal person who is answerable for court directions and orders against the foreign litigant, or

2. The foreign party has status to sue in its home jurisdiction

INTERNATIONAL ASSN. OF SCIENCE AND TECHNOLOGY V. HAMZA (1995) ABCA

F: Matrimonial property dispute b/w Mr. & Mrs. Hamza. 2 Swiss associations sought declaration that neither Mr. H or

2

Page 3: cans.allardlss.comcans.allardlss.com/.../cans/Edinger_76_Fall_2017_Jasmin…  · Web viewkConflict of Laws 2017. Some General Considerations. Keep in mind that while activities being

Mrs. H had any legal or equitable interest in the assets held by them. Mrs. H argues that Ps do not have standing in AB to seek such a declaration (b/c not legal persons under AB law—which limits standing to natural and legal persons). I: Can Ps have standing in AB, despite fact that they are not recognized as legal persons in AB? L: The decision whether or not to recognize standing of a foreign entity must be made in light of the principle of comity. Generally, forum should look to whether the entity is a “juridical person” by law of country where it is domiciled. If recognized as such by the foreign jurisdiction (and the forum is satisfied that court directions and judgements are enforceable against entity in foreign jurisdiction) then will have requisite characteristics to gain status to sue in Canada (even though similar entity domiciled in forum would not have status). R: The rule of what entities can be a party to an action is procedural (in control of forum) but can be supplemented by law of the entity’s home jurisdiction on basis of principal of comity. A: The court looked at laws applying to AB residents/corps., under which Ps would not have had legal status to sue b/c not legal or natural person. But, b/c Ps are not resident in AB, req’d to look at AB’s conflict rules. Guidance from s. 282(1) of AB’s Bus. Corp. Act, which curtails legal action of unregistered or extra-provincial corporations wrt to K actions in AB—court interprets this as a limitation not on status but on specific kind of action, meaning other actions would be allowed. General rule that an entity can sue but must prove they’re recognized as juridical person as home jurisdiction. Courts recognize all foreign entities with legal status to sue under law of domicile. This approach strengthened by principle of comity. C: Ps has status to sue on triable issue.

BUMPER DEVELOPMENT CORP LTD V. COMMISSIONER OF POLICE OF THE METROPOLIS (1991) ENGLISH CA

F: Indian citizen digs up bronze statue and sells it. Eventually it ends up in hands of Bumper Development. Police seize statue in London and return to India. Bumper sues police for return of statue. Other parties include India, a ruined temple, and a sacred stone (both located in India).I: Do stone and temple have standing to join the action? L: Under Hindu law, which is recognized by Indian law, temple has status to sue. A: Court looks to home jurisdiction re status—easy to locate the temple, which is in India. It is a legal person under law of Tamil Nadu, Indian state where it is situated. Court reasoned that comity (except as limited by forum PP) means parties having standing under foreign law be given standing in the forum to enforce rights. C: Yes, temple has status to sue (no need to bother with stone).

EXCLUSIONARY RULES

Relevant to laws selected under Choice of Law rules and Recognition & Enforcement Proceedings. Exclusionary rules apply to actions but also to settlement judgements (USA v. Harden, SCC 1963). You only invoke exclusionary rules (as defence) if foreign lex causa.

General Exclusionary Rules (Dicey)1. English courts do not have jurisdiction to entertain an action for enforcement, directly or indirectly, of a penal,

revenue, or other public law of a foreign state (characterization issue)2. English courts will not enforce or recognize a right, power, disability, or legal relationship arising under the law

of a foreign country if the enforcement or recognition of such a law would be inconsistent with the fundamental public policy of English laws.

PENAL

Forum will not enforce the laws of a foreign sovereign in its sovereign capacity, and this is why we don’t enforce the penal laws of a foreign jurisdiction. Same reason we don’t enforce revenue laws of FJ directly or indirectly.

3

Page 4: cans.allardlss.comcans.allardlss.com/.../cans/Edinger_76_Fall_2017_Jasmin…  · Web viewkConflict of Laws 2017. Some General Considerations. Keep in mind that while activities being

Law: The forum court will not enforce the penal laws of another state (Huntington, 1893 PC), meaning: A proceeding by a state whose laws have been infringed, or by a member of the public as a “common

informer” Penal = Must be a suit in favour of the state whose law has been has been infringed—if penalty is sought for

the state, an official, or individual on behalf of the whole community, then it comes within the rule. Could be a civil or criminal law to be considered penal.

HUNGTINGON V. ATTRILL, 1893 PRIVY COUNCIL

F: Under NY statute, Corp. officers are liable for corp. debts to creditors if they make false representations. H creditor of NY corp., A is officer of that Corp. H sues A in NY on basis of above statute. H awarded $100k judgement against A. A doesn’t have any assets in NY and doesn’t pay. H brings R&E action in ON, where A does have assets. A argues the NY judgement is based on penal law and not enforceable in Canada. I: Is this judgement based on penal law? How does forum decide if law in question should be characterized as penal or not? What does “penal” even MEAN? L: A law is penal when suit is in favour of state whose law has been breached. R: Forum court determines whether a law is penal based on its own definition of penal. In Canada, penal proceeding by state or public against a party. A: Forum is not bound by characterization of law by foreign jurisdiction. Should carefully consider how a foreign court would characterize it, but ultimately, forum must decide whether a law is penal by application of its own definition.). “Penal” includes any proceeding in nature of a suit by the state (or an officer of the state or member of the public on behalf of community as a whole aka “public informer”) whose law has been infringed for recovery of pecuniary penalties, and all judgements for pecuniary penalties.H: NY law not penal, but protective and remedial. It is a civil remedy only available to corporate creditors (not enforceable by state or public gen.). Ontario does not view violations of laws that regulate trading as offences against the state for conflict purposes. Judgement is R&E against Attrill.

REVENUE

*Rule covers all forms of taxation at any level of government. Law: Courts won’t enforce foreign revenue laws (i.e. a claim to pay foreign taxes) (Harden, SCC 1963)

o Applies to all taxes at any level of government o Applies to direct and indirect enforcement (court will not entertain an action brought by an indiv’l

which will have indirectly the effect of enforcing revenue laws of FJ).

STRINGHAM V. DUBOIS, 1992 ABCA

F: D dies domiciled in Arizona. Left her AB for to S (Niece). Executor in Arizona wants to sell AB farm to pay US estate taxes owed to US gov’t. S sues in AB for order of full conveyance and tax exclusion. Executor counter claims to force sale. I: Does an order for the sale of the farm breach exclusionary rule for revenue laws? H: Estate tax is a foreign revenue claim. Court allows conveyance and disallows sale to pay US taxes. R: USA v. Harden (1963 SCC) is leading case in Canada re rev excl. rule. Courts will not entertain an action brought by an individual that will directly or indirectly enforce revenue laws of a foreign country. A: Court considers three important cases:

Hardoon v Belilios, [1901] AC 118 (Privy Council)o Cestui que trust who gets all the benefit of the property should bear its burden, unless they show

some good reason why trustee should bear the burden.o Obligation of cestui que trust to indemnify trustee against calls upon them indisputable, unless

there’s some K or other circumstance excluding the obligation. USA v Harden, [1963] SCR 366

o Courts won’t entertain action brought by a foreign government or individual which will, directly or

4

Page 5: cans.allardlss.comcans.allardlss.com/.../cans/Edinger_76_Fall_2017_Jasmin…  · Web viewkConflict of Laws 2017. Some General Considerations. Keep in mind that while activities being

indirectly, have the effect of enforcing revenue laws of a foreign countryo Two explanations for rule against enforcing foreign tax judgments offered:

1) enforcement would be an extension of the sovereign power which imposed taxes, contrary to concepts of independent sovereignty, and

2) court won’t recognize liabilities of foreign state if they run counter to “settled public policy” of its own—public policy always on reserve and state relations shouldn’t be determined by courts.

Re Reid (1970), 17 DLR (3d) 199 (BCCA) Insufficient assets in UK to pay UK taxes, UK law made trustee responsible for taxes.

o Trustee paid tax, then sought reimbursement with Hardoon rule and a statute.o B opposed on grounds that Harden rule was sufficiently good reason to oust Hardoon and statute.o Court refused to apply Harden and distinguished it on two grounds: 1) in this case, the state isn’t

enriched, and 2) in each case state was a party, but here UK has nothing to do with the claimCourt finds 1st reason that Reid distinguished from Harden unpersuasive (the state is enriched by the taxes paid by the trustee) and in any event in the case at hand AZ/US state is enriched. Also disagrees with suggestion of 2nd ground that the state needs to be a party to the action. For these reasons, Re Reid can’t be followed. Court comments that SCC may want to revisit the problem in the current international context of comity etc., but till then Harden is binding. Also notes that the US taxing authority has recourse against S b/c she is a US resident.C: Enforcement would be indirect enforcement of AZ revenue laws.

PUBLIC POLICY

Law: A court will not enforce laws that breach forum public policy (very narrow in the conflicts context):o “fundamental values”, “essential principles of justice” (Society of Lloyd’s v Meinzer, 2001 ONCA)o It would be “offensive to participate in enforcement of the judgment” (Boardwalk Regency)o “It would violate some fundamental principle of justice, some prevalent conception of good morals,

some deep-rooted tradition of the common weal” (Kuwait Airlines quoting Loucks)Conceptions of what breaches forum public policy are constantly evolving

o Ex. gambling debts never used to be enforced, now sometimes will be enforced

SOCIETY OF LLOYDS V. MEINZER (2001) ONCA

F: M were private investors with Lloyd’s, where a K stipulated England as choice of forum and law. Investment exposed Pl’s to unlimited liability, refused to pay. England held M obliged to pay. L brings R&E action in Ontario. M argued R&E of English judgments would indirectly violate Ontario’s public policy (not filing a prospectus according to Securities Act). H: enforces English judgments, defense fails. A: Protecting public in trading is a fundamental value, wouldn’t have been entertained in Ontario…BUT b/c of history of litigation in ON, estopped from no R7E b/c would undermine whole process of respecting the clause, staying the ON action and sending to England for decision. [Edinger disagrees that noncompliance with Securities Act would breach public policy.] Fundamental values at play with Securities Act, but two factors weigh in favour of R&E: 1) history of litigation in ON; and 2) principles of international comity – public policy of enforcing rules of comity where justice, necessity and convenience all favour enforcement outweighs the public policy concerns wrt Securities Act.

R: public policy reflects a “local sense of justice and public welfare”, “it must violate some fundamental principles of justice”, essential morality, or “deep-rooted tradition” of the forum (e.g. fraud, bribery, coercion, prostitution). Look to historical and factual context of proceedings leading to foreign judgment, ask whether R&E is contrary to public policy. PP: high threshold reached here, but R &E justified by comity, recognize international K’s made by Lloyd’s.

KUWAIT AIRWAYS CORP. V. IRAQI AIRWAYS CO (NO. 4&5) (2002) ENGLAND HL

F: Iraq invaded Kuwait. 10 planes seized, taken back to Iraq, and law passed dissolving Kuwait Airlines. Title on planes passes to Iraqi Airways. Kuwait regains sovereignty, Iraq has to repeal law dissolving Kuwait airlines, but Iraqi

5

Page 6: cans.allardlss.comcans.allardlss.com/.../cans/Edinger_76_Fall_2017_Jasmin…  · Web viewkConflict of Laws 2017. Some General Considerations. Keep in mind that while activities being

Airways keeps planes. Action in tort for conversion brought by KA in England. English CL rule for transfer of title of movable property inter vivos depends on the law of the place where

the assets were located at the time the transfer occurred: in this case, Iraq. KA says court should refuse application of Iraqi law with respect to whether 1) tort actionable there and 2)

law that purported to transfer title to IA b/c breach forum PP.I: Are there PP grounds on which forum court should refuse to recognize and apply the Iraqi laws?R: Laws that fundamentally breach IL or infringe on HR are examples of laws that may be contrary to public policy in the conflicts context. L: Yes. Expropriatory decree was made under unacceptable purposes/circumstances. Gross violation of established rules of int’l law. A: Normally COL rule of UK would imply application of Iraqi law. However, courts retain residual power to exercise discretion to disregard foreign law when to do otherwise would be affront to basic principles of justice and fairness which courts seek to apply. This discretion must be exercised exceptionally and with caution. Counsel for IA argued breach of Forum PP limited to cases involving breach of Human Rights, but HL rejects this. Find could also occur in circumstances where fundamental breach of international law. C: Iraqi law in this circumstance was contrary to PP of forum b/c breached IL. Therefore, forum refuses to apply Iraqi law. Note: Definition by Cardozo J: “the court will exclude the foreign decree only when it “would violate some fundamental principle of justice, some prevalent exception of good morals, some deep-rooted tradition of common weal.” Court also says acceptability of provision of foreign law “must be judged by contemporary standards…conceptions of PP should move with the times.”

Where PP exception is invoked to exclude the application of foreign law, usual result is that forum court will apply its own law, although in Kuwait court just ignored offending provision and otherwise applied the foreign law.

PP definition for purpose of conflicts SUPER NARROW—PP invoked all the time, rarely succeeds.

OTHER PUBLIC LAW

Law: Courts have no jurisdiction to entertain an action for the enforcement, either directly or indirectly, of a penal, revenue, or “other public law” of a foreign state (Dicey)

o “Laws will not be enforced if they involve an exercise by a government of its sovereign authority over property beyond its territory” (AG of NZ v Ortiz)

Examples (no cases of it succeeding in Canada):o Export, automatic forfeiture provisions (NZ law unenforceable in England)o Official Secrets Act, maintenance of national security (UK law unenforceable in Aus)

US V IVEY (1995 ONT GEN DIV) – RESIDUAL DISCRETION, OTHER PUBLIC POLICY

F: Michigan gov’t seeks to enforce 2 money judgments on Ontario defendants, which ordered them to pay cost of remedial measures for a waste disposal operation in Michigan (polluter pays laws). R&E action in Ontario. Defendants try 4 defences (jurisdiction and 3 exclusionary rules). H: No defences apply. Cleanup costs not penal as it is not a criminal penalty. Not revenue as the law’s purpose is to have those who create a hazard bear cost of cleanup. Not public law as it defendants merely pay for cleanup costs, not an attempt by a foreign state (US) to assert its sovereignty in Ontario. R: “public law”, forum can’t enforce law of a political or public nature, or a governmental interest of a foreign country (exchange, control, export legislation). It is a residual category and narrowly defined, hard to invoke, but does exist in Canada. EE: use only if desperate

6

Page 7: cans.allardlss.comcans.allardlss.com/.../cans/Edinger_76_Fall_2017_Jasmin…  · Web viewkConflict of Laws 2017. Some General Considerations. Keep in mind that while activities being

DOMICILE & RESIDENCE

Relevant to laws selected under CL Choice of Law rules and to jurisdiction. Onus of proof rests on party alleging a change in domicile. Deals with relationship of an entity

DOMICILE

Standard of proof for domicile used to be close to beyond a reasonable doubt, now proven on BoP (requires clear and cogent evidence).

A person can only possess one domicile at any given time. There are three types:

1. Domicile of Origina. Test: Place where your father domiciled at the time of your birth (qualifications: if father is

unknown, then place your mother was domiciled at time of your birth, if neither parent known, then wherever you were found as infant. Has been displaced in some instances with statute).

2. Domicile of Dependencya. Test: Until you reach the age of majority, place you’re your caregivers are domiciled (Infant’s Act).

i. Traditionally CL based on father’s domicileii. It also used to apply to married women, as well as minors and mentally incompetent. Law

& Equity Act = no longer applicable to married women, spouses considered independent in terms of domicile.

3. Domicile of Choicea. Test to Acquire: Once you reach age of majority, place where you physically reside with freely

informed intent to remain there indefinitely (Agulian). i. If you intend to return to domicile of origin in event of reasonably likely to occur

circumstances you do not acquire domicile of choice. If return is only vague possibility, however, a domicile of choice may be acquired.

1. Probability of return assessed from perspective of persion; 2. Focus on intent to remain in DoC, not on intent to return to DoO.

ii. Statement of intent to acquire new domicile of choice not sufficient on its own (Foote). Requires physical presence coupled with intent.

b. Test to Abandon: cease to reside there coupled with either intent never to return to residence there (Foote) or without an intention to return.

i. Choice to leave must be voluntary and not due to circumstances such as business, debts, health, etc.

ii. If you abandon domicile of choice without acquiring a new one, domicile reverts to DoO. iii. In BC b/c we have unreformed CL concept of domicile, probably requires leaving DoC with

formed intent never to return.

PRESENCE + INTENT TO REMAIN INDEFINITELY

7

Page 8: cans.allardlss.comcans.allardlss.com/.../cans/Edinger_76_Fall_2017_Jasmin…  · Web viewkConflict of Laws 2017. Some General Considerations. Keep in mind that while activities being

AGULIAN V. CYGANIK (2006) EWC (ENG CA)

F: A died in London in 2003 w/estate worth $6.5 M. Left $50 K to partner C in will that was made in 1995 (2 years after they got together). C applied for variation of will and more $ under UK statute. Eligibility depends on whether A domiciled in England or Cyprus at death.

A had been born in Cyprus, and moved to UK at 18 and lived in UK till his death But maintained strong ties to Cyprus: he returned frequently, tried to start a business there, owned

property, sent his daughter to live there. Evidence was that he only left b/c of an engagement gone bad where jilted in laws threatened his life.

Parties agree A domicile din Cyprus prior to ’95. But where domiciled after ’95 contested. I: Was A domiciled in England or Cyprus at time of death? L: Domicile of Origin can only be displaced by Domicile of Choice where clear intent to remain indefinitely. Burden of proof on party alleging a change in domicile. Cannot underestimate enduring strength of DoO, must consider the whole of person’s life in order to determine whether there was intent to make their home permanently/indefinitely in new place. A: DoO prevails in absence of DoC. DoC takes hold where presence is coupled with clear intent to remain indefinitely in a place. Question of whether DoO is displaced by DoC is one of fact. DoC acquired only if can be shown that person has formed independent intent to reside indefinitely. Its incorrect to focus on whether A had intent to return to Cyprus, rather, should focus on whether he had formed the intent to live permanently in England. Too much weight was places on the last few years of A’s life. Must look to the whole of the person’s life. C: TJ decision overturned, A domicile din Cyprus, not England. Lady is shit out of luck re will variation.

Re Fuld, 1968 Fuld was born in Germany, sent to England for school, interned in Canada, got Canadian citizenship and law

degree in Canada Divided his life b/w Germany, Canada, and England (could not make up his mind where he wanted to settle

down) When can’t form necessary intention or settle on DoC, then DoO revives. Court finds Fuld’s domicile when he died was DoO Germany.

CEASE TO RESIDE + CESSATION IN INTENTION

FOOTE V. FOOTE ESTATE (2011) ABCA

F: EF born in AB. Lawyer, married with a bunch of kids. Divorces 1st wife. Goes into business distributing cleaning products. Quits the law and travels the world. Marries two more times. Eventually makes his home in Norfolk Isl. (tax haven) starting in 1972. He also buys a condo in Victoria BC near end of life and makes noise about moving there one day. Gets ill, dies in AB where he went for health treatment. $220M in assets in BC, AB, Aus, British Virgin Islands, etc. He leaves three wills to deal with his various kids and current and former wives. Each will had “poison pill” clause, try to contest directly/indirectly only entitled to portion of residue of estate. Current wife and five children of first marriage contemplating applying for relief, would have to be pretty sure of success though if they challenged b/c of poison pill clauses. Make application to court to determine domicile of EF on his death. They say EF domiciled in Canada at death, either b/c acquired new DoC in BC, or abandoned DoC in Norfolk and reverted to DoO AB. TJ finds that EF DoO Alberta but acquired DoC in Norfolk Isl. I: Where was EF domiciled at time of death? R: Choice to change domicile must be voluntary and not dictated by business, debt or health. Intent to change DoC is not, on its own, sufficient to change DoC. A: Q of Law and Fact, so standard of review is palpable and overriding error. Did TJ err? CA thinks not. Evidence put forward that EF had requisite intent to change DoC insufficient. Ultimate deciding factor ofr the court is that EF didn’t get any tax advice about moving, which would have been a step he’d definitely have taken b/f changing domicile considering significance of his assets. Demonstrates he hand’t formed requisite intent to change domicile in BC or abandon domicile in NI. C: Domiciled in NI on death.

National Trust Co. Ltd. V. Ebro, 1954 England HC Domicile of corporation is place in which it was incorporated.

8

Page 9: cans.allardlss.comcans.allardlss.com/.../cans/Edinger_76_Fall_2017_Jasmin…  · Web viewkConflict of Laws 2017. Some General Considerations. Keep in mind that while activities being

RESIDENCE

A person can have multiple ordinary residences (and likely habitual as well) (per Knowles) Residence of a corporation is wherever that corporation is carrying on business (so corp could be multiply

resident even though only has one domicile). Intent could be a factor to establish residence, but not as important here as it is to domicile Three types of Residence of concern in conflicts:

1. Actual: Place where you physically are at given time. 2. Ordinary: Physical presence for an extended and regular basis, coupled iwht intent to live there on a

more or less regular basis (Nafie) Where your life is centred; where you make your home and reside in course of ordinary, day to day

life. Can be maintained when absent from the place for weeks, months, years (e.g. army service) Some passage of time in the actual place required, but no minimum established. Used in BC for purposes of jurisdiction (see CJPTA 3(d)).

3. Habitual: Residence adopted voluntarily and for settled purposes, with physical presence that endures for some time (PA v KA).

In theory, somewhat more permanent that ordinary residence. Depends on quality of time spent, not length of time. Some passage of time in place required, but no established minimum. “settled purpose” could be specific or general, time limited or not. Used in BC for purposes of jurisdiction (FLA) and choice of law rules.

Residency is treated as a spectrum

ORDINARY RESIDENCE

NAFIE V. BADAWY (2015) ABCA

F: N&B destination wedding in 200 in Egypt. Return to Calgary. 2011 B moved to Saudi Arabia for 1 yr K job. N & 3 kids joined 2 weeks later. Leased AB home, lease home in SA and two cars, and enrol kids in US run school, also opened bank account. Maintained ties to AB: kept their house in Calgary, kept 1 of 3 cars, kept bank accounts, cell #s, health insurance, received mail and continue to pay Canadian income tax. June 2012 N returns to AB and moved in with parents. B says 2-week vacation and N had return tickets and interviewing for job in SA while in AB. N said this was an act to escape B. She stays in AN and files for divorce one month later. One spouse needs to be OR in province for min. 1 year to file for divorce. TJ says N was, despite 8 mo. Physical absence in pretending year. I: What does ordinary resident mean? Was N ordinarily resident in AB for one year preceding application? R/L: Ordinary residence is where one’s life is centred, where one makes their home for current time. A: TJ erred by overemphasizing N’s intent. Take care looking at intent which may change after the fact. While statute may define ordinary residence, in absence of definition it is where party’s home was for the time being. Where they settled into and maintained ordinary mode of living. It is fact specific and matter of degree—contrasts with casual/temporary/occasional—different from a stay or visit. OR indicates settled in, maintained mode of life. Court looks at cases and finds that OR can be maintained even when absent from OR for weeks, months, and in one case years (but last exceptional, was serving in military). C: Ordinary residence in SA. (Dissent says SA always meant to be temporary set up and TJ saying OR was AB was reasonable).

2 ORDINARY RESIDENCES POSSIBLE

KNOWLES V. LINDSTROM (2014) ONCA

F: A & R lived in FL 2002-2012, separated in Feb. A flew to Toronto and applied for spousal support. R said no jurisdiction and even if so, FL forum conveniens. In the alternative, FL applies and A has no claim. I: Is it possible to have more than one ordinary residence? What about primary residence (in this case, FL)?L: It is possible to have more than one ordinary residence; if the statute makes no mention of primary residence then ordinary residence is sufficient. A: Both FL and ON are ordinary residences because they spent substantial time each year in both places. Ordinary residence is the presumptive connecting factor for litigation – if the statute is silent about primary residence, then

9

Page 10: cans.allardlss.comcans.allardlss.com/.../cans/Edinger_76_Fall_2017_Jasmin…  · Web viewkConflict of Laws 2017. Some General Considerations. Keep in mind that while activities being

ordinary residence is good enough to warrant ON’s jurisdiction over the claim. C: Ordinary residence in ON established and ON has jurisdiction.

HABITUAL RESIDECE | QUALITY OF RESIDENCE + PHYSICAL PRESENCE FOR SOME TIME

PA V. KA (1987) ABCA

F: Wife US then Canadian citizenship. Husband Canadian. Married in AB in 1970 and lived there 14 years. Sept 1982 wife lived in Hawaii for 3 months, reconciled with husband and lived in AB 14 months, then both went to Hawaii in March 1984. Got alien cards for employment, sold some things, took some things. Lived with friends. Daughter went to school there. Wife got a job, husband didn’t. May 84 moved into house with month-to-month tenancy. Mid-May husband returned to AB on biz, including disposal of point assets. End of June he returned to Hawaii for 6 weeks, then Cali in mid-August, then OR. Told wife he wanted a divorce in Oct 84. Wife started divorce and matrimonial property action May 85 in AB. Divorced Oct 85. F: Where was their last joint and habitual residence? (Does AB have jurisdiction with respect to property division?)L: Habitual Residence refers to quality of residence (not duration, although can be a factorhj) with physical presence that endures for some time A: Intent isn’t as important as for domicile, but may be a factor (present intent, not future intent). ‘Habitual” means the residence was adopted voluntarily and for “settled purposes” – purpose could be for limited period, could be specific or general. Husband didn’t establish ties to Hawaii, even though he intended to. C: Joint and habitual residence AB.

JURISDICTION IN PERSONAM

Jurisdiction in actions in personam = actions whose purpose is to impose a personal obligation on the defendant owed to the plaintiff (e.g. damaged for breach of K). At common law, two components to question of jurisdiction:

1. Are the rules satisfied? (if so there is jurisdiction simpliciter/territorial competence)2. Discretion (even if juris. Simp., court may choose not to take jurisdiction and hear action).

Canada has developed rules about assumed jurisdiction which are independent of the procedural rules about service of the originating process (service still important for other reasons, but no longer basis for jurisdiction in Canada).

THE CONSTITUTIONAL STANDARD

It has been argued that constitutional doctrine restricts the extraterritorial reach of provincial law generally, and that the entire body of conflict of laws principles---such as those dealing with the jurisdiction of the courts, the recognition of extra-provincial judgements, and choice of law—must be evaluated according to a constitutional standard. Taken up in Morguard from which we get principle that in the context of taking jurisdiction, the courts are constitutionally restrained so that they can assume jurisdiction only where a real and substantial connection exists b/w the province in question and the dispute. Morgaurd principle typically arises in situations where there is service ex juris.

Common law test: For jurisdiction to be properly assumed, there must be a real and substantial connection between the forum and the action (Morguard)

o Real = not hypothetical, substantial = not weak (Van Breda)

Real and substantial connection under CJPTA:10 Without limiting … a real and substantial connection between BC and [the facts on which a proceeding is based] is presumed to exist if the proceeding

(a) Relates to rights or interest in property in BC, (b) Concerns administration of the estate of a deceased person in relation to

(i) Immovable property in BC of the deceased person, or (ii) Movable property anywhere if the deceased person was ordinarily resident in BC

(c) Relates to a deed, will, contract or other instrument related to

10

Page 11: cans.allardlss.comcans.allardlss.com/.../cans/Edinger_76_Fall_2017_Jasmin…  · Web viewkConflict of Laws 2017. Some General Considerations. Keep in mind that while activities being

(i) Property in BC, or(ii) Movable property anywhere if deceased person was ordinarily resident in BC

(d) Is brought against a trustee in relation to(i) Relief related to trust assets that are property in BC,(ii) Trustee is ordinarily resident in BC,(iii) Administration of trust principally carried on in BC,(iv) Express terms of trust state [the] trust is governed by BC law

(e) Concerns contractual obligations, and (i) They were to be performed in BC to a substantial extent,(ii) K is governed by law of BC by its express terms,(iii) K

(A) Is for purchase of property, services or both, for use other than in course of purchaser’s trade or profession, and

(B) Resulted from solicitation of business in BC by or on behalf of the seller(f) Concerns restitutionary obligations that arose in BC to a substantial extent(g) Concerns tort committed in BC(h) Concerns business carried on in BC(i) Is a claim for an injunction to do or refrain from doing anything

(i) in BC, or(ii) in relation to property in BC

(j) Is to determine personal status or capacity of a person ordinarily resident in BC(k) Is for enforcement of a judgment or arbitral award made in or outside BC, or(l) Is for recovery of taxes or other debt and is brought by BC gov’t or local authority

MORGUARD INVESTMENTS LTD. V DE SAVOYE (1990) SCC

*Majority Decision written by La Forest J.

*Case imposes constitutional standard on conflict of law matter: Reasonable & Substantial Cxn Req’d.

F: D’s owed money to MIL under a mortgage on property in AB. Ds moved to BC by the time they defaulted on the mortgage, and had no other assets in AB. MIL brings action in AB, served Ds in BC, but Ds take no steps to defend action in AB. No clause in mortgage agreement that AB agreed jurisdiction. MIL gets default judgement in AB including for $$ not covered by sale of foreclosed property, seeks to have enforced in BC where Ds now live and have assets. Under CL rules, BC courts could not enforce the judgement (Ds were not present in AB and did not submit to jurisdiction of AB courts). Still BC courts decided to enforce the judgements. Ds appealed to SCC. I: Can a personal judgement validly given in one province against an absent D be enforced in province where D now resides? R: The standard for jurisdiction simpliciter is that there is a real and substantial connection between the action and the court. A: Jurisdiction is territorial, a state’s law has no binding effect outside its jurisdiction. English approach rigidly applied this doctrine, and was incorporated into Canadian law. Modern states can’t live in such “splendid isolation” (we’re globalized).

Modern states recognize judgements of other states sometimes, stemming from principle of comity (aka deference and respect due by other states to the actions of a state legitimately taken with in its territory). La Forest J. say really comity not just about deference, it’s about convenience, NAY necessity given the division of legal authority amongst sovereign states and need to facilitate the flow of wealth, skills, people across borders in globalized world with attention to principles of order and fairness. For short, combo of duty & convenience (Hilton v Guyot, 1894 USA as cited in R v. Spencer (1985) SCC).

The considerations underlying rules of comity apply with much greater force b/w units of a federal state. A regime of mutual recognition of judgements across the country is inherent in a federation. “…in a federation…a fuller and more generous acceptance of the judgements of the courts of other constitution units…” is necessary, and “must rules of comity and private international law as they apply b/w the provinces “must be shaped to confirm to the federal structure of the constitution.”

Courts in one province should give “full faith and credit” to judgments of another, provided the court exercised jurisdiction properly and appropriately. A court can exercise jurisdiction against an out-of-province D for an in personam judgment where there is a real and substantial connection between the action and the jurisdiction. Here, the land was in AB, mortgage was granted in AB, MIL & DS in AB when mortgage was made – so there is a R&SCxn.

11

Page 12: cans.allardlss.comcans.allardlss.com/.../cans/Edinger_76_Fall_2017_Jasmin…  · Web viewkConflict of Laws 2017. Some General Considerations. Keep in mind that while activities being

C: Judgment R&E b/c there is R&SCxn b/w action and AB.NB: La Forest J. notes that no difficulty re “real and substantial connection” where D was within jurisdiction at time of action or when submitted. Difficulty is where D served ex juris, as was the case here. When considering whether R&SCxn asking what strength of nexus is b/w subject matter of action and territory where action is brought. Here R&SCxn pretty obvious, properties in AB and Ks regarding properties entered into in AB.

Spar Aerospace, 2002 SCC

One basis in QB rules for QB taking jurisdiction was that damage was suffered in QB Damage to corporation in QB amounted to $50 Does $50 of damage fulfill the “substantial” requirement from the “real and substantial” standard?

o Not at issue in the case but it would meet the standard for QB to have jurisdiction.

CLUB RESORTS V. VAN BREDA (2012) SCC

*Majority written by Lebel J.

F: Mr. V signed K to give tennis lessons at resort in Cuba in exchange for room and board for himself and Mrs. V. She was working out at resort one day when machine collapsed causing her to become quadriplegic. Tort action commenced by her against resort in Ontario. Hotel was owned by company domiciles in Cayman Isl. Club argued Ontario lacked jurisdiction and, in the alternative, that Cuba more appropriate based on Forum non conveniens. I: Does Ontario have jurisdiction? R/L: R&SCxn means connection b/w state and dispute can’t be weak or hypothetical. A: There is a distinction b/w private international law and constitutional dimensions of R&SCxn Test.

In constitutional sense, places limits on the reach of the jurisdiction of province’s courts and on the application of provincial laws to interprovincial or international situations. Also req’s all Canadian courts R&E decisions of other Canadian courts where jurisdiction was properly assumed.

Constitutionally imposed territorial limits on jurisdiction are related to, but distinct from, R&SCxn test in conflicts.

o Conflicts rules include: when jurisdiction can be assumed, what law governs, and R&E.o Constitutional territorial limits include: setting outer boundaries w/in which variety of conflicts rules

can be applied, makes sure conflicts rules remain w/in boundaries and so authorizes assumption of jurisdiction only in circumstances with legitimate exercise of state’s power of adjudication.

R&SCxn b/w state and dispute can’t be weak or hypothetical, as would cast doubt upon legitimacy of the exercise of state power over the person affected by dispute.

C: R&SCxn b/w ON and action b/c K was entered into in Ontario. NB: Not weak ≠ substantial so seems like Van Breda is setting a pretty low bar re req’d cxn. Connective factors are objective factors connecting legal situation or subject matter of litigation with forum. In case of tort, presumptive (rebuttable) connective factors include:the defendant is domiciled or resident in the province; the defendant carries on business in the province; the tort was committed in the province; and a contract connected with the dispute was made in the province. Also note that forum non conveniens argument failed. Trial in Cuba would have posed serious challenges.

PARTIES WITHIN THE JURISDICTION

Plaintiff can be anywhere (submits to jurisdiction by starting the action there) so forum concerned with location of the defendant. Traditionally, CL has held that mere physical presence was sufficient to give court’s jurisdiction over D as of right (see Maharanee), aka has territorial competence. Van Breda holds traditional grounds under CL still valid, physical presence will give the court jurisdiction as of right (PP being that territory has right to control any person physically present within the territory)(affirmed in Chevron). Exception to CL rule is if D was tricked into coming into forum’s jurisdiction.

12

Page 13: cans.allardlss.comcans.allardlss.com/.../cans/Edinger_76_Fall_2017_Jasmin…  · Web viewkConflict of Laws 2017. Some General Considerations. Keep in mind that while activities being

Test for territorial competence: Individuals: D was served within the jurisdiction (Maharanee) Out of province corp: D was present + carried on business in jurisdiction at time of action (Chevron).

o Requires some actual (i.e. not virtual) presence in the jurisdiction (Van Breda, Chevron).

MAHARANEE OF BARODA V WILDENSTEIN (1972) CA ENGLAND

F: P lives in France but also lived in England for extended periods, she owned houses and flats there. D is an art dealer who lives in France. P purchased $33K artwork allegedly by Boucher, D gave certificate of value for “Boucher”. P tried to auction it later and was told it was not a Boucher, and worth $500 at best. P lawyers issue writ against D, wait until D next came to England to serve him. D asked service to be set aside. I: Does English court have jurisdiction simpliciter?R: Mere presence of D in the forum (England) is sufficient to give the (English) court jurisdiction as of right. A: (Lord Denning) Even though P, D both live in France, P did no wrong serving D in England. He was voluntarily in England (not tricked), and properly served in England so court’s jurisdiction is validly invoked. Jurisdictional rules are satisfied if you find the party within the jurisdiction and serve them. C: Service valid.

CHEVRON CORP V YAIGUAJE (2015) SCC

F: 1972-90 Texaco involved in joint venture with government of Ecuador, lots of environmental damage done. Texaco made a big payment to Ecuadorian government to help clean up the mess, but it wasn’t spent on that. 1993 action from Ecuadorian Ps in NY. 1995 Texaco persuades NY to stay action b/c EC forum conveniens. NY requires D to submit to EC jurisdiction, agrees to on condition it’s permitted to defend against any judgment from EC. 2001 Chevron buys Texaco. 2003 Ps recommence action in EC. TJ grants $18B in damages, reduced to $9B on appeal in 2011. 2014ish Chevron sues P’s lawyer in NY for writing judgment, buying Ws, etc. NY agrees with Chevron, upheld on appeal. 2012 Ps sue Chevron and Chevron Canada in ON for R&E of EC judgment. Chevron has no presence in ON but has many subsidiaries, including CC (7th generation subsidiary). CC argued it wasn’t a party to the judgment and ON had no jurisdiction over it. R/L: Physical presence + carrying on biz gives court jurisdiction. A: R&SCxn doesn’t oust traditional grounds giving jurisdiction (D’s presence, or submission). To establish presence-based jurisdiction over an out-of-province corporate D, must show that D was “carrying on biz” in the jurisdiction at the time of action. Does the corp have some direct or indirect presence in the state asserting jurisdiction, accompanied by degree of biz activity which is sustained for a period of time? CC is present in ON, carrying on biz there and had a place of biz there. Some kind of physical presence required. CC has actual office in ON. If there’s physical presence, don’t need to consider R&SCxn (for R&E?). CC may not be liable, but action can proceed against them. C: Ct has jurisdiction over CC.NB: Just b/c jurisdiction is established does not mean Ps will succeed in R7E of EC judgement—just gives them the opportunity to seek R&E.

PARTIES OUTSIDE THE JURISDICTION

How does a court assume jurisdiction over a defendant not in the jurisdiction (service Ex Juris)?1. Is the application being brought in a province that has enacted a version of the CJPTA (BC, Nova Scotia, & Saskatchewan)? Choose yes or no and determine whether court has jurisdiction simpliciter aka territorial competence.

Yes- CJPTA in force (BC used as example) No-CJPTA not in force

13

Page 14: cans.allardlss.comcans.allardlss.com/.../cans/Edinger_76_Fall_2017_Jasmin…  · Web viewkConflict of Laws 2017. Some General Considerations. Keep in mind that while activities being

1. Can the court assume jurisdiction ex juris?

s. 2(2)

Territorial competence of [BC] court is determined solely by reference to [part 2 of CJPTA]

s. 3 A court has territorial competence in a proceeding that is brought against a person only if:

a) that person is the P in another proceeding in the court to which the proceeding in Q is a counterclaim;

b) during the course of the proceeding submits to court’s jurisdiction;

c) there is an agreement b/w the P and that person to effect that the court has jurisdiction in the proceeding;

d) that person is ordinarily resident in BC at time of commencement of proceeding, or

e) there is a R&S cnxn b/w BC and the facts on which the proceeding against the person are based.

s. 10 [This is a non-exhaustive list of circumstances that = R&S Cxn]

R&S Cxn presumed if proceeding:(a) is brought to enforce, assert, declare or

determine proprietary or possessory rights or a security interest in property in British Columbia that is immovable or movable property,

(b) concerns the administration of the estate of a deceased person in relation toi. immovable property in British

Columbia of the deceased person, or

ii. movable property anywhere of the deceased person if at the time of death he or she was ordinarily resident in British Columbia,

(c) is brought to interpret, rectify, set aside or enforce any deed, will, contract or other instrument in relation toi. property in British Columbia that

is immovable or movable property, or

ii. movable property anywhere of a deceased person who at the time of death was ordinarily resident in British Columbia,

(d) is brought against a trustee in relation to the carrying out of a trust in any of the following circumstances:i. the trust assets include property

in British Columbia that is immovable or movable property

1. Can the court assume jurisdiction ex juris?

The court can assume jurisdiction where there is a real & substantial cnxn b/w defendant and action (Morguard).

The R&S required for assumed jurisdiction must be found in each case through a presumptive connecting factor (Van Breda).

Onus to establish PCF on plaintiff (Van Breda).

If PCF established, Defendant has opportunity to rebut. The test to rebut an established PCF is whether the D has established facts “that demonstrate that the PCF does not point to any real relationship b/w the subject of the litigation and the forum, or points only to a weak relationship b/w them (Van Breda).

If no PCF established OR PCF established but D successfully rebuts, then NO assumed jurisdiction.

14

Page 15: cans.allardlss.comcans.allardlss.com/.../cans/Edinger_76_Fall_2017_Jasmin…  · Web viewkConflict of Laws 2017. Some General Considerations. Keep in mind that while activities being

and the relief claimed is only as to that property;

ii. that trustee is ordinarily resident in British Columbia;

iii. the administration of the trust is principally carried on in British Columbia;

iv. by the express terms of a trust document, the trust is governed by the law of British Columbia,

(e) concerns contractual obligations, andi. the contractual obligations, to a

substantial extent, were to be performed in British Columbia,

ii. by its express terms, the contract is governed by the law of British Columbia, or

iii. the contract(A) is for the purchase of

property, services or both, for use other than in the course of the purchaser's trade or profession, and

(B) resulted from a solicitation of business in British Columbia by or on behalf of the seller,

(f) concerns restitutionary obligations that, to a substantial extent, arose in British Columbia,

(g) concerns a tort committed in British Columbia,

(h) concerns a business carried on in British Columbia,

(i) is a claim for an injunction ordering a party to do or refrain from doing anythingi. in British Columbia, orii. in relation to property in British

Columbia that is immovable or movable property,

(j) is for a determination of the personal status or capacity of a person who is ordinarily resident in British Columbia,

(k) is for enforcement of a judgment of a court made in or outside British Columbia or an arbitral award made in or outside British Columbia, or

(l) is for the recovery of taxes or other indebtedness and is brought by the government of British Columbia or by a local authority in British Columbia.

MORAN V. PYLE NATIONAL LTD., 1973 SCC

*Note: this is pre-Morguard, deals with location of tort only for jurisdictional purposes (don’t use for COL)

F: Light manufacturer in ON makes defective light bulb, parts are from various places Bulb purchased and installed in SK, man who installing electrocuted and dies. His widow bring action in SK under Fatal Accident Act, but for Act to apply, tort must have occurred in SK.

I: What rule should be used to locate a tort of negligent manufacture for jurisdictional purposes? R: To determine if court has jurisdiction over defendant in tort action dealing with negligent manufacture, ask whether D could reasonably foresee that product would be used or consumed by P in

15

Page 16: cans.allardlss.comcans.allardlss.com/.../cans/Edinger_76_Fall_2017_Jasmin…  · Web viewkConflict of Laws 2017. Some General Considerations. Keep in mind that while activities being

place P used or consumed it. L: Dickson J. adopts a flexible rule. Acknowledges that for jurisdictional purposes, possible that tort located in more than one jurisdiction (not to be confused with location of tort for choice of law purposes!). Question is D knows or ought to know that tort may occur in place that plaintiff is, then the forum in which P suffered damage in is entitled to exercise judicial jurisdiction over that foreign defendant. C: SK has jurisdiction.

CLUB RESORTS LTD. V. VAN BREDA, 2012 SCC

*Dealing with tortF: HB has tennis K with resort, wife injured in resort gym. I: How should the court determine whether there is a R&S Cxn b/w tort action and forum allowing court to assume jurisdiction over foreign D? R: R&S Cxn requires P to establish a presumptive connecting factor b/w facts of case and forum. For tort actions presumptive factors include: a) D is domiciled or resident in forum; b) D carries on business in forum; c) tort committed in the province; or d) K connected with dispute was made in the province. non-exhaustive. L: Question of territorial competence (jurisdiction simpliciter) is an initial and separate question from decision of whether or not court will exercise its discretion to hear a matter (if no jurisdiction simpliciter, no discretion to exercise).

The overarching consideration for determining jurisdiction simpliciter is whether there is a R&S Cxn b/w facts and forum (constitutional principle, limits reach of provinces and courts).

However, that is broad consideration, framework req’d to determine what = R&S Cxn in order to provide certainty and predictability. In otherwords, a list of objective factors that will establish R&S Cxn:

o Does not have to be uniform across Canada (legislation can change it)o Informed by principles of fairness, efficiency, and comityo Presumptive (thus rebuttable)o Plaintiff must establish 1+ factoro Burden on D to rebut any established factors

List of PCF in case of torts:o D is domicile or residence in forumo D carries on business in forum (must be actual presence of some sort, not virtual!)o Tort committed in forumo K connected with dispute was made in province

Presence of D in forum alone not PCF on its own. Fact that damages were sustained in forum alone not sufficient. Nor is combined effect of factors that are non-connecting.

Relevant considerations in identifying new factors:o Similarity of CF with recognized PCFo Treatment of CF in the case lawo Treatment of the CF in statute lawo Treatment of CF in private int’l law or other legal systems with shared commitment to order,

fairness, and comity If P does not establish a PCF (recognized or new), then court should not take jurisdiction If P does establish PCF, D can rebut the presumption of jurisdiction

o D must establish facts that demonstrate the PCF does not point to any real relationship or only a weak relationship b/w the subject matter of action and the forum.

E.g. K made in prov. That had little to do with subject matter or litigation, or subject matter of litigation has little to do with D’s business in jurisdiction.

If D successfully rebuts, no jurisdiction If P established PCF and D either doesn’t rebut or fails in rebuttle, court has jurisdiction and can then decide

whether it should exercise its discretion to decline jurisdiction if forum non conveniences is raised by one of the parties.

C: ON has jurisdiction on basis that K for Mr. V to provide tennis lessons made in ONNB: This is the one that waters down substantial connection from Beals because they use all the negative phrases “not hypothetical, not weak” which is not the same threshold as substantial.

16

Page 17: cans.allardlss.comcans.allardlss.com/.../cans/Edinger_76_Fall_2017_Jasmin…  · Web viewkConflict of Laws 2017. Some General Considerations. Keep in mind that while activities being

COURT V. DEBAIE, 2012 ABQB 640

*TortF: Ps are AB residents suing relative for defamation. Ds lived in Ab at time of alleged events but now life elsewhere in Canada.

Pleadings failed to allege that publication had occurred in AB (need publication in AB in order for tort of defamation to have occurred in AB)

Ds asked for action to be dismissed on basis that AB did not have jurisdiction. I: What support is sufficient to establish a PCF (tort occurred in jurisdiction) to meet the R&S Cxn standard for service outside of the jurisdiction? R: In establishing a PCF for service ex juris, P need only make out a “good, arguable case” for the factor (e.g. can rely on hearsay to establish PCF). L: Court will accept the assertions/evidence as true and ask if based on this, the P has a good, arguable case. Standard of Proof that would apply at trial does not apply in context of establishing jurisdiction. A: Ps brought affidavits with hearsay evidence that someone read the posts in AB, court says this is sufficient to establish PCF. NB: Be sure to include jurisdictional facts as well as evidence on substantive merits of the case.

TAMMINGA V. TAMMINGA, 2014 ONSC

*TortF: P, resident of ON, travels to farm in AB where she is injured, returns to ON

Commences action in ON against relative, corporation, and insurance company AB defendants argue no jurisdiction simpliciter and/or forum non-conveniens P argues that ON has jurisdiction, PCF is that her insurance K was made in Ontario (Van Breda)

I: Is the Insurance K a sufficient PCF to give ON jurisdiction over non-resident defendants? R: A K must be sufficiently connected ot the dispute to raise the presumption of a R&S cxn to the jurisdiction. A/C: K of insurance anticipated accidents generally but did not have anything to do with accident in AB (nothing that connects the insurance K to the Ds, P did not visit AB for any reason connected to K).

JTG MANAGEMENT SERVICES V. BANK OF NANJING, 2015 BCCA

*ContractF: JTG exporting lumber to purchaser relying on letter fo credit from Bank in Shanghai that states bank will pay JTG for exported lumber when received required documents form JTG

JTG provides documents, Bank says they aren’t good enough and won’t pay. JTG sues Bank in BC, service ex juris.

Bank argues that BC does not have territorial competence JTG argues that BC has territorial competence

I: How to determine whether BC has jurisdiction simpliciter/territorial competence over K dispute?R: To determine if a K is to be performed to a substantial extent in the jurisdiction so as to give court jurisdiction simpliciter/territorial competence under the R&S test, the court must look at the K in its entirety and the nature of the obligations that arise under it. L: More than one court may properly assert jurisdiction over parties’ dispute b/c K can involve obligations in more than one jurisdiction.

CJPTA s. 10(e) says BC has jurisdiction if action concerns K obligations and those were to be performed in BC to a substantial extent.

To determine whether K is to be performed “to a substantial extent in BC, court must look at the K in its entirety and the particular nature of the obligations that arise under the K.

A/C: This K had obligation to present documents in China and obligation to make payment in BC, both are

17

Page 18: cans.allardlss.comcans.allardlss.com/.../cans/Edinger_76_Fall_2017_Jasmin…  · Web viewkConflict of Laws 2017. Some General Considerations. Keep in mind that while activities being

substantial portions of the K. Receipt of payment constitutes substantial portion of K obligations, which was to be performed in BC, therefor jurisdiction under CJPTA s. 10(e)(i). NB: CJPTA does not have provision for Ks made in province (removed by ULC on basis that does not meet R&S test).

CHEVRON CORP. V YAIGUAJE, 2015 SCC

F: Chevron served ex juris (parent company not present in ON). Chevron argues that ON does not have jurisdiction simpliciter for R&E of foreign judgement. I: What is req’d for court to take jurisdiction simpliciter over D for purpose of R&E of a foreign judgement? R: An exception to Morguard/Van Breda: Don’t need a R&S Cxn b/w forum and D if the action is for R&E of a foreign judgement. L: In order for court to take jurisdiction in case of R&E of foreign judgement, only requirement is that defendant is serve (which will be service ex juris). Rationale:

Difference b/w action at first instance and action for R&E is that only purpose of action for R&E is to allow a pre-existing obligation to be fulfilled (not to determine substantive merits of the claim)

Notion of comity (militates in favour of generous enforcement rules)A/C: Chevron was served ex juris pursuant to ON rules of court therefore ON has jurisdiction for the action to R&E foreign judgement. NB: Still open to D to make argument that forum court should not enforce the judgement b/c a) foreign court did not have jurisdiction to make order. Can also argue that ON should not exercise its jurisdiction on basis of forum non conveniens. Could also argue exclusionary rule as defence (e.g. fraud, Public policy). Chevron only argues that ON does not have jurisdiction simpliciter and does not argue that they do but should

not exercise discretion b/c other CL courts would see this as submission to ONs jurisdiction and would enforce ON judgement.

Per Edinger this is problematic b/c ignores the constitutional standard—there is no R&S Cxn in this case. o Hopefully impact of Chevron will be controlled b/c few judgement creditors would bring an action in

jurisdiction where D has no presence or assets.

LAPOINTE ROSENSTEIN MARCHAND MELANCON LLP V. CASSELS BROCK & BLACKWELL LLP, 2016 SCC

F: Cassels Brock was primary firm involved with GM closing dealerships (via wind down agreements b/w GM and dealerships).

Condition of agreement was that each dealership get independent legal advice Lass action started by dealerships against CB for negligence in providing legal advice CB joined all the firms that had provided ILA to dealerships, including many outside ON Some QB firms argued that ON did not have jurisdiction (over them, didn’t want to be joined)

I: Is there a R&S Cxn b/w ON and the cause of action against the 3rd party law firms? L: Van Breda: There is a R&S Cxn if the K is made in ON (one of presumptive connecting factors).

First step is to identify the dispute Then determine whether a K connected with dispute was made in the jurisdiction (sufficient

that dispute be connected to a K made in the province where assumption of jurisdiction proposed)

A: Wind down agreement is the K connected to the dispute, connected CB to 3rd party claims against local

lawyers K made in ON, therefore ON had jurisdiction

D: (Cote J) says re Application: One, the wind down agreements were made in QB (acceptance occurred in QB); and two, the wind down

agreement is not connected with the dispute (QB lawyers were not brought w/in contractual relationship b/w GM and dealership, the QB firms’ obligations flow from their own retainer agreements)

18

Page 19: cans.allardlss.comcans.allardlss.com/.../cans/Edinger_76_Fall_2017_Jasmin…  · Web viewkConflict of Laws 2017. Some General Considerations. Keep in mind that while activities being

NB: Per Edinger: SCC diminishes R&S connection to virtually nothing here.

* If court lacks jurisdiction, the proceeding will be dismissed.

2. If Jurisdiction Simpliciter not established, proceeding halts there. However, where jurisdiction simpliciter established, Court still has discretion to refuse to hear the application

- Discretion (more important of elements between discretion and jurisdiction simpliciter) can take two forms o Stay the local proceedings (in favour of some other jurisdiction)o Anti-suit injunction

Prohibits other party from continuing their action in the other jurisdiction More aggressive of the two remedies (so harder to obtain, courts are cautious in ordering

these) But courts are more likely to grant anti-suit injunction when there is a contract action

with a contract that has a jurisdiction selecting clause in it (will protect its own jurisdiction then)

Per Alchem:

“The courts have developed two forms of remedy to control the parties' choice of forum. The first and more conventional device is a stay of proceedings. It enables the court of the forum selected by the plaintiff (the domestic forum) to stay the action at the request of the defendant if persuaded that the case should be tried elsewhere. The second is the anti-suit injunction which may be granted by the domestic court at the request of a defendant, actual or potential, in a foreign suit. With the stay, the domestic court determines for itself whether in the circumstances it should take jurisdiction whereas, with the injunction, it in effect determines the matter for the foreign court. While the restraining order operates in personam on the plaintiff in the foreign suit and not on the foreign court itself, it has the latter effect and therefore raises serious issues of comity. A court should entertain an application for an anti-suit injunction only where a serious injustice will occur because of the failure of a foreign court to decline jurisdiction applying the forum non conveniens test.” Whether court has taken jurisdiction on basis of presence, submission, or service ex juris, obligation on D

to argue forum non conveniens and request a stay in proceedings (Van Breda) SCC confirmed forum non conveniens as part of Canadian Law in Alchem Products Inc v. British Columbia

(Worker’s Compensation Board), 1993 but rejected the two-step analysis adopted in English Spiliada. Common Law test in Canada, as stated in Alchem: A stay is to be granted when there is a clearly more

appropriate forum for resolution of the Dispute. Rationale behind doctrine: gives defendants a counterweight against broad forum choice afforded to

plaintiffs—restrains Ps from choosing forum with little real connection to the dispute simply to gain some advantage. It also allows disputes to be resolved in forum with which there is the strongest connection. Last, key to holding parties to K agreements re choice of forum.

Where Forum Non-Conveniens is before the court, burden of proof is always on the D (Van Breda).

Forum Non-Conveniens

CJPTA Applies Common Law Applies

*BOP on D (ED says in class notes)

s. 11 1) After considering the interests of the parties to a proceeding and the ends of justice, a court may decline to exercise its territorial competence in the proceeding on the ground that a court of another state is a more appropriate forum in which to hear the proceeding.

2) A court, in deciding the question of whether it or a court outside British Columbia is the more appropriate forum in which to hear a proceeding, must

Court will not decline to exercise properly assumed jurisdiction unless the D invokes forum non-conveniens or an anti-suit injunction.

Forum Non-Conveniens Test: is there some other

forum more convenient and appropriate for the pursuit of the action and for securing the ends of justice (Alchem)

19

Page 20: cans.allardlss.comcans.allardlss.com/.../cans/Edinger_76_Fall_2017_Jasmin…  · Web viewkConflict of Laws 2017. Some General Considerations. Keep in mind that while activities being

consider the circumstances relevant to the proceeding, including:- the comparative convenience and

expense for the parties to the proceeding and for their witnesses, in litigating in the court or in any alternative forum,

- the law to be applied to issues in the proceeding,

- the desirability of avoiding multiplicity of legal proceedings,

- the desirability of avoiding conflicting decisions in different courts,

- the enforcement of an eventual judgment, and

- the fair and efficient working of the Canadian legal system as a whole.

Burden of proof is on the D to show why the court should decline to exercise its jurisdiction, this will be through showing evidence of strong connecting factors pointing to other court as more convenient (unless statute specifies otherwise) (per Van Breda).

o You can borrow factor list from CJPTA framework…

If Court finds there is another forum conveniens, and declares itself forum non-conveniens issues a stay of proceedings.

Anti-Suit Injunction

Forum court not to entertain if no foreign proceeding pending. Party must have also tried to obtain stay in foreign court (note court can be persuaded to ignore if not

dealing with CL system in foreign Jurisdiction b/c civil systems don’t have judicial discretion). Then ask, is forum court alleged to be the natural forum for the action, and is it potentially the most

appropriate forum? (If not, will not entertain application). Then ask: is there another forum that is clearly more appropriate than the domestic forum?

o Comity means domestic court considers, applying own rules relating to forum non-conveniens, the foreign court could reasonably have concluded that it was forum conveniens (and no other alternative forum that was clearly more appropriate).

o If FC decision reasonable Anti-suit injunction stayed BUT if DC concludes that FC assumed jurisdiction on basis inconsistent with principles relating to forum non

conveniens, and that the FC’s conclusion could not reasonably have been reached if it had appropriately applied those principles, then ask:

o Will an injustice result if the P is allowed to proceed in foreign jurisdiction? o Will an injunction unjustly deprive the P of advantages in the foreign forum, with regard to the

extent that the party and facts are connected to it? (weigh loss of advantage b/w D and P, where little cnxn b/w action and forum, P probably not losing any advantage injunction being issued).

o Based on outcome of this weighing injunction either granted or dismissed.

AMCHEM PRODUCTS INC. V. BC (WCB), (1993) SCC

F: Ps (mostly BC residents) injured by asbestos exposure; Ds are American corporations. - BCWCB commences action in Texas (where courts have lots of experience with this particular issue)- Ds requested that Texas stay action based on forum non conveniens, but were unsuccessful- Ds ask BC court to issue anti-suit injunction to prohibit continuance of Texas action

I: How does a Canadian court consider forum non-conveniens?

- How does a Canadian court decide whether to grant an anti-suit injunction where there is a foreign proceeding underway?

R: Forum Non-Conveniens- Key question (whether service is ex-juris or not) is whether there is a more appropriate forum for the issue

to be heard, based on the relevant factors?o BoP is on the D—regardless of basis that court assumed jurisdiction on—to demonstrate that the

domestic forum is not the most appropriate forum (unless statute says otherwise). o The existence of a more appropriate forum must be clearly established to displace the forum

selected by the P. Anti-Suit Injunction

20

Page 21: cans.allardlss.comcans.allardlss.com/.../cans/Edinger_76_Fall_2017_Jasmin…  · Web viewkConflict of Laws 2017. Some General Considerations. Keep in mind that while activities being

Applicant must show the following:- There is a foreign proceeding pending; - There was an unsuccessful attempt to get the foreign court to stay its proceedings; and - The forum court is alleged to be the most appropriate forum, and is potentially most appropriate forum

(run through forum non-conveniens test here). If above two criteria met, Domestic court will engage in 2-step analysis:

o Step 1: In applying the forum non conveniens principles that we adhere to in Canada (relevant factors point to court that has closest connection with the action and parties)—could Foreign court have reasonably concluded that it was forum conveniens? (does not have to be the most appropriate, as long as no other potentially connected forum is not more clearly appropriate—in this case there were cnxns to texas and other possible forums not clearly more appropriate than texas).

If yes, respect decisions of foreign court and application dismissed. If no, move on to step 2.

o Step 2: Court will consider general injustice to P if injunction granted, and general injustice to D if P is allowed to proceed with foreign proceedings. In general, court will not grant an injunction if, by doing so, it will unjustly deprive P of advantages in foreign forum. But a party can have no reasonable expectation of advantage in juris. Where party and subject of litigation have little or no connection.

Another way to deal with this question is to ask: Would D’s in foreign action be deprived of some juridical advantage that = injustice if Ps allowed to proceed. (not Q of whether “oppressive or vexatious” because this has no legal meaning here).

C/H: No anti-suit injunction issued. Was reasonable for Texas court to assume jurisdiction in the circumstances. NB: Sopinka J “With due respect to the trial judge, the principle of comity to which I have referred does not require that the decision of the foreign court be based on the doctrine of forum non conveniens...It is the result of the decision when measured against our principles that is important and not necessarily the reasoning that leads to that decision.”

This test may be a problem in civil law jurisdictions that are rule based and don’t have concept of exercising discretion (ex: can’t ask court in civil law jurisdiction to exercise discretion to stay their own proceeding)

Canadian version of discretion generally follows Spilliada except on burden of proof wrt forum non conveniens and the procedure for obtaining anti-suit injunctions

TECK COMINCO METALS LTD. V. LLOYDS UNDERWRITERS, 2009 SCC

This is a CJPTA case on appeal from BC dealing with Forum Non ConveniensF: Teck sued Lloyds for insurance coverage for enviro damage that Teck caused in WA

- Teck started action in WA, WA refuses to stay its proceedings- Lloyds commences parallel proceedings in BC- Teck applies to BC court to have BC proceeding stayed- BCSC and BCCA refuse to stay the BC action

I: What does the CJPTA mean wrt whether a BC court should stay the BC proceedings in light of the prior parallel proceeding in WA and the assertion of jurisdiction by the WA state court? R: CJPTA has codified the CL forum non-conveniens test—apply that analysis as normal

- Fact of parallel action is just one factor to consider amongst others, it is not determinativeL: S. 11 of CJPTA applies even when foreign court has asserted jurisdiction

- S. 11 is a complete codification of the CL FNC test- Prior assertion of jurisdiction by a foreign court does not oust the s. 11 inquiry- The fact that there is a parallel action pending is not an overriding and determinative factor in s. 11 of the

analysis:o Avoidance of multiplicity is one of several factors to be considered in s. 11 analysiso Policy reason for this: adopting this as determinative actor would encourage a first to file system

(parties would rush to commence proceedings in jurisdiction it wants; considerations that have little to do with where action most appropriately heard would carry the day).

A: TJ considered all relevant factors under s. 11, therefor no reason to overturn TJ’s decision (no stay of BC Action).

21

Page 22: cans.allardlss.comcans.allardlss.com/.../cans/Edinger_76_Fall_2017_Jasmin…  · Web viewkConflict of Laws 2017. Some General Considerations. Keep in mind that while activities being

CLUB RESORTS LTD V. VAN BREDA, 2012 SCC

CL Case re Forum Non-Conveniens on appeal from OntarioF: The shitty hotel gym equipmentR: Once court has jurisdiction, should only consider forum non conveniens if the defendant raises it. L: Normal state of affairs = jurisdiction should be exercised once properly assumed

- Court will not decline to exercise its jurisdiction unless D invokes forum non-conveniens

Court will consider discretion/forum non-conveniens if the defendant invokes it- BOP on D to show why the court should decline to exercise its jurisdiction- D must clearly id a more appropriate forum (can’t just say somewhere else)

o Follow same rules re R&S cxn with local forum to show cxn b/w alternative forum and the action. - Examples of factors—see CJPTA s. 11 which codifies CL factors

Forum non conveniens is designed to temper the application of jurisdiction—CL makes jurisdiction simpliciter wide and will narrow it if someone invokes discretion. NB: BC courts do best to incorporate SCC decisions even if based on CL and CJPTA (e.g. CJPTA does not use term clearly, but BC requires appropriateness of another forum to be clearly established).

JURISDICTION SELECTING CLAUSES

3 Different Approaches to Jurisdiction Selection Clauses:1. Disregard and declare void (contrary to PP, early CL approach)2. Give them great weight (current CL approach)3. Treat them as absolute (approach usually take by statute)

At CL, 3 issues may arise wrt Jurisdiction selecting clauses1. What does the clause cover?

a. Interpretation issue does the clause extend to this particular action? 2. Is the clase valid?3. What is the effect that CL gives to such clauses in K?

a. CJPTA s. 11 does not address jurisdiction selecting clauses. This is a problem b/c Teck says that s. 11 is exhaustive. But Huyde (SKCA) says that you read Pompey test into CJPTA legislation. Under Pompey, BoP reversed (D argues FNC and asks for stay based on juris-selection clause, BoP on P to demonstrate why shouldn’t be held to JSC).

POMPEY INDUSTRE V ECU-LINE N.V., 2003 SCC

Leading case F: K had exclusive jurisdiction selecting clause choosing Belgium (“K is gov’d by law of B and any claims or disputes arising under K shall be determined by the courts in Antwerp and no other court”).

- Cargo being transported damaged, P allege cargo damaged while in transit and being handled by Ds. - P (Pompey) commences action against ECU (D) in Canadian Federal Court, ECU applies for stay of

proceeding based on jurisdiction selecting clause in K (bill of lading).I: What is the proper test for a stay of proceedings to enforce a forum selection clause? Does the test contemplate an inquiry into whether there was a fundamental breach/deviation, or should that inquiry be left to the decision maker in agreed upon forum? R: Once court determines that a validly concluded K binds the parties, court must grant the stay unless P can show sufficiently strong reasons to support conclusion that it would not be reasonable or just in circumstances to require the P to adhere to forum selection clause. Court must take into account all the circumstances of the particular case in exercising discretion. Court is not to consider whether fundamental breach of K. L: Exercise of discretion to stay the proceeding to enforce a jurisdiction selecting clause is the “strong cause” test:

- Courts must give weight to desirability of holding K parties to their agreement

22

Page 23: cans.allardlss.comcans.allardlss.com/.../cans/Edinger_76_Fall_2017_Jasmin…  · Web viewkConflict of Laws 2017. Some General Considerations. Keep in mind that while activities being

- BOP on plaintiff to prove there is good reason why P should not be bound by forum selecting clause- Consider same factors as in Amchem: convenience of parties, fairness b/w parties, interests of justice. (but

this is different test than when there is an application for stay w/ no jurisdiction selecting clause due to the weight that must be given to agmt)

In application for a stay to uphold a jurisdiction selecting clause in a K, the court must not consider w/ one party has deviated from/fundamentally breached the K (to determine whether breach prevents party from relying on exclusion clause).

- This will be determined by proper law of the K (law chosen to govern K). A: Factors in Favour of Appellant: prefers to litigate in a familiar jurisdiction and does not bring up the jurisdiction clause merely to seek a procedural advantage; there are reasonable connections with Belgium; there are Belgian and French witnesses; any time bar which may preclude the respondents from bringing their case in Belgium has been waived; no security has been posted; and the enforcement of a Belgian judgment against the appellant should present no particular difficulties. Factors in Favour of Respondent: accepted the respondents’ arguments that there will be Canadian and American witnesses in these proceedings; the Tribunal de commerce in Antwerp conducts its proceedings in Flemish and decides cases on the basis of documents and statements, a procedure precluding witnesses and cross-examination; and, there may be more delay in Belgium than in Canada, especially if there is an appeal. C: The prothonotary concluded that the factors in favour of denying a stay, while substantial, were just short of the “strong cause” test which the respondents had the burden of meeting. I see no reason why the prothonotary’s conclusion on this point should be set aside.

MOMENTOUS.CA CORP. V. CANADIAN AMERICAN ASSOCIATION OF PROFESSIONAL BASEBALL LTD., 2012 SCC

follows PompeyF: Professional baseball team played one season in Ottawa, then wanted to leave

- League rejected their application to withdraw - Team sued the league in Ontario; league issued statement of defence- League asked for dismissal of action on basis of jurisdiction selecting clause selecting North Carolina

I: What test governs when a party asks for a dismissal of the action because of a jurisdiction selecting clause when that party has issued a statement of defence in the contested jurisdiction?R: Strong cause test applies to application for dismissal based on jurisdiction selecting clause even if party has issued a statement of defence in the contested jurisdictionL: Ontario rules of court do not require that application for a stay due to a forum selecting clause be brought before delivering a statement of defence

- Proper test to apply is the “strong cause” test – unless there is a strong cause as to why a domestic court should exercise jurisdiction, order and fairness are better achieved when parties are held to their bargains

A: No strong cause here (not even argued).

HUYDE FARMS INC. V. CANADIAN WHEAT BOARD, 2011 SKCA

CJPTA jurisdiction F: Contract between parties with jurisdiction selecting clause giving MB courts exclusive jurisdiction. Huyde Farms brought action against Wheat Board in SK; Board applied for stay of SK action.I: How does “strong cause” test from Pompey for application for stay apply in context of jurisdiction selecting clause where CJPTA governs?R: Test for application for stay when there is a jurisdiction selecting clause in a CJPTA jurisdiction is the same as the test in Pompey (read Pompey into the CJPTA (B.C.’s s. 11))L: Court must first determine if it has territorial competence

- -Question as to the appropriate forum does not arise unless the territory where an action has been commenced, has territorial competence over the action

- Court considers whether to give effect to the forum selecting clause as part of exercise of determining whether

23

Page 24: cans.allardlss.comcans.allardlss.com/.../cans/Edinger_76_Fall_2017_Jasmin…  · Web viewkConflict of Laws 2017. Some General Considerations. Keep in mind that while activities being

jurisdiction should be declined pursuant to (B.C.’s) s. 11 of the CJPTA- While CJPTA does not have reference to forum selecting clause, Pompey/strong cause test applies- Burden on the plaintiff to demonstrate a strong cause not to enforce the clause

A: Huyde Farms have not demonstrated strong cause therefore stay SK action Commentary: Huyde sets out 4 step framework when defendant applies under CJPTA for order that Court decline competence over an action:

(1) Does the province have territorial competence over the matter? If no, action cannot continue (2) If yes, has the defendant/applicant established that forum selection clause is valid, clear and enforceable and that it applies to the action before the court? If no, application fails(3) If yes, has the plaintiff/respondent shown strong cause for why the court should not give effect to the forum selecting clause? (If yes, defendant may be able to demonstrate some other basis for court to decline competence)(4) If no, the Court should consider whether it is appropriate to transfer the action to another territory pursuant to Part III of the CJPTA

DOUEZ V. FACEBOOK, 2017 SCC

CJPTA jurisdictionF: K between FB and user with jurisdiction selecting clause selecting California. Douez brings class action in BC, relying on BC Privacy Act; FB objects b/c we had a JSC in K!I: How is an application for a stay on the basis of a forum selection clause treated in B.C. wrt the CJPTA?R: SCC upholds BCCA. In B.C., in an application for a stay on the basis of a forum selection clause in a commercial K, the Pompey test applies as standalone inquiry; CJPTA analyzed secondly if necessary. SCC also says that FNC and forum selection clauses to be treated separately, and s. 4 of Privacy act does not, and is not, intended to invalidate forum selection clauses. L: In B.C., when defendant relies on forum selection clause, the Pompey test applies as a separate, standalone inquiry that is conducted first; the CJPTA analysis may be conducted second, if necessary

-Pompey requirement that plaintiff show clause is valid, clear and enforceable and applies to the action does not impose evidentiary burden on defendant (defendant does not need to adduce expert evidence indicating that forum would have territorial competence under its own law) - In B.C., when a defendant does not rely on a forum selecting clause, the analytical framework of s. 11 of the CJPTA applies

A: FB showed clause to be valid, clear and enforceable; assumed that clause applied - S. 4 of Privacy Act does not override the forum selection clause - Douez did not demonstrate strong cause for Court not to enforce the clause - Do not need to go to CJPTA- Enters stay

COMMENTARY: Based on SCC results, hard to say what outcome will be in situation of a consumer contract.

Additional notes

- No jurisdiction selecting clause: burden of proof on the defendant - Contract action and contract has jurisdiction selecting clause that is valid and enforceable:

o B.C.: Pompey/CL analysis (outside CJPTA), burden on plaintiff to show strong cause for why action should continue in B.C.

o SK: Pompey analysis as part of CJPTA, burden on plaintiff to show strong cause for why action should continue in SK

24

Page 25: cans.allardlss.comcans.allardlss.com/.../cans/Edinger_76_Fall_2017_Jasmin…  · Web viewkConflict of Laws 2017. Some General Considerations. Keep in mind that while activities being

RECOGNITION & ENFORCEMENT OF IN PERSONAM JUDGEMENTS

While used together, Recognition and Enforcement are separate steps. Recognition is the process by which the court accepts the validity of the decision on its merits and that it has resolved the issue b/w the parties. Enforcement is the 2nd step. It involves lending assistance to a party to follow through on the judgement. All foreign judgements must be classified as either in personam or in rem.

In Personam : are judgements as against another legal person. They can be pecuniary or non-pecuniary. o Pecuniary judgements award damages ($$$)o Non-pecuniary/equitable are orders to do or refrain from doing something (e.g. an injunction, order

for specific performance). In Rem : References judgements with respect to property or status. Connected to property, status, and

maritime claims. Territorial sovereignty requires that foreign judgements be converted into a local judgement to be enforceable in the jurisdiction. Historically, CL only enforced pecuniary judgements, in Canada courts may now appropriately recognize & enforce non-pecuniary judgments as well. Two methods of judgement conversion in BC:

1. CL method of R&Ea. Can apply to any judgement from anywhere.

2. Statutory Method (the choice of which will depend on the origin of the foreign judgement):1

b. Court Order Enforcement Act, Part 2i. Limited to judgements from jurisdiction w/ mutual recognition

agreement with BCc. Enforcement of Canadian judgements & Decrees Act

i. Applies to every Canadian province & territory, regardless of whether agmt w/ BC or not.

The advantage of the statutory methods is that all you have to do is register the foreign judgement to which it applies and R&E achieved unless judgement debtor objects. R&E under CL requires commencement of an action. Limitation Periods apply to having foreign judgements R&E by the court.

BC limitation is 10 years or the foreign limitation period, whichever is shorter, to commence an action (Limitation Act s. 7(b)).

AB and ON have limitation period of 2 years to commence an action

BC Rules for when BC Court Will R&E in personam judgement

In an action on a foreign judgement, the test for whether a court will recognize and enforce the foreign judgement as three requirements: 1) it must be final; 2) the court granting the judgement must have had jurisdiction on a particular basis (jurisdiction in the international sense); 3) for enforcement, the judgement must be for a fixed sum of money and not a take or penalty or must be a non-monetary judgement that the court is willing to be enforced (aka exclusionary rule does not apply and if it is non-pecuniary, meets req’ments). Chevron has negated the necessity of the forum court in Canada establishing jurisdiction over the defendant in order to R&E a foreign judgment. The court basically said that basis for taking jurisdiction is found simply and wholly in the defendant being served with process. So, basically diametrically opposed to Morguard where they said service of process cannot form basis of jurisdiction. But the court justifies this by saying that considerations underlying substantive claims are different than those in defending enforcement proceedings. End result = with no threshold jurisdiction requirement for an enforcement action, judgement debtor defendants can now be required to advance and establish defences in a forum that may have no connection with the judgment.

Requirements for R&E

(1) Foreign judgment must be final and conclusive, and

1 If the judgement is from within Canada, go directly to the Enforcement of Canadian Judgements & Decrees Act. If it is from outside of Canada, and there is an agmt b/w BC and Jurisdiction, you can use the Court Order Enforcement Act, Part 2. If neither of these options work you must apply CL rules.

25

Page 26: cans.allardlss.comcans.allardlss.com/.../cans/Edinger_76_Fall_2017_Jasmin…  · Web viewkConflict of Laws 2017. Some General Considerations. Keep in mind that while activities being

(2) The foreign court must have had jurisdiction over the action in the international sense

“In the international sense” = jurisdiction according to our (English) rules Three ways that court can find that foreign court had jurisdiction

o (i) Presence Presence in the jurisdiction at the time the action was commenced (service in the originating

jurisdiction) Forbes v. Simmons

o (ii) Submission, or By the defendant to the jurisdiction of the foreign court

o (iii) Real and substantial connection Between the action and the jurisdiction Morguard, Beals v. Saldanha

o Note: Morguard and Beals have both held that the traditional CL rules (presence and submission) are still relevant

If the above requirements are met, Defendant can still rely on CL defenses that remain operational. These include: Breach of natural justice; fraud; and the exclusionary rules. What advice to give client who is being sued in foreign jurisdiction

- Is presence met (was client served in the foreign jurisdiction)?o If yes, then jurisdiction in international sense met and judgment will be R&E.

- If not presence, probably don’t give advice to not do anything to avoid submission because enforcing court may find real and substantial connection with originating court after the fact.

- So, advise client to make an appearance o Argue that foreign court does not have jurisdiction

If successful, then no action and client wins! o If B.C. client, can probably argue in foreign CL court that the foreign court is not the most appropriate

court for the action If successful, then no action there!

o If client loses on both of those grounds, then client should think seriously about entering a defense on the merits because good chance judgment against them will be R&E by enforcing court in jurisdiction where client has assets on basis of real and substantial connection.

PECUNIARY JUDGEMENTS

1) Finality and Conclusiveness

NOUVION V. FREEMAN (1889) HL

F: Claim in England against deceased’s estate. Claim based on a Spanish judgement. I: Does a foreign judgement have to be final and conclusive for R&E in forum court? If so, how is it determined whether order is F&C? R: The foreign judgement must be F&C for forum court to R&E it. At CL, burden on plaintiff to establish finality judgement they seek to have R&E. To determine whether or not a judgement is final and conclusive, determine whether the foreign judgement is res judicata (cannot be pursued any further).

Can the parties go back to the same court to vary the order or obtain a new judgement? If so (can have trail de novo) then it’s not final and conclusive But note ability to appeal passes F&C muster (although in Canada would be very common for defendant to

apply for stay in forum juris while issue on appeal in foreign jurisdiction, and would nearly always be granted in BC pursuant to rule 19-3(8)).

*Examples of orders that are not final and inclusive in Canada include support/maintenance orders, orders respecting care of children. These orders cannot be enforced at common law, therefor require statutory authority to enforce.2) Jurisdiction in the International Sense

26

Page 27: cans.allardlss.comcans.allardlss.com/.../cans/Edinger_76_Fall_2017_Jasmin…  · Web viewkConflict of Laws 2017. Some General Considerations. Keep in mind that while activities being

(i) Presence: Mere physical presence is sufficient to establish jurisdiction of the foreign court in the international sense

Presence needs to be considered as of a particular time—specifically, when the proceedings were commenced. Presence at some earlier time, e.g. when K was made, is not sufficient. Nor will something like citizenship suffice. It doesn’t matter, however, how fleeting or brief presence is so long as it overlaps with proceedings commencing.

For Corporate presence: carrying on business in the jurisdiction = presence (office is enough). But court will apply its own test for “carrying on business” not the test of the foreign jurisdiction.

E.g. Chevron—Chevron Canada carrying on business in Ontario, presence in Ontario established.

FORBES V. SIMMONS, 1914 ABSC

F: D an AB resident visiting ailing wife in BC hospital when served for BC action. P succeeds in BC action. BC plaintiff commences action in AB for R&E of BC judgement. D argues BC court did not have jurisdiction therefore unenforceable in AB. I: What is presence for the purpose of jurisdiction in the international sense when it comes to R&E? R: Physical presence, even fleeting, at time you are served = presence at CL for R&E purposes.

(ii) Submission

Submission must be voluntary. Some forms of duress may negate submission. Submission can occur either through appearance or contract. If party appears in foreign court and defends case on the merits (anything beyond arguing no jurisdiction) then party has voluntarily submitted to the jurisdiction.

FIRST NAT’L BANK OF HOUSTON V. HOUSTON E&C INC. (1990) BCCA

F: Action for R&E of Texas judgement in BC Court. D’s argued that they did not submit. However, they had hired a lawyer in Texas who had made an appearance. I: How is submission to foreign court’s jurisdiction determined? R: Appearing without protest in foreign action is considered voluntary submission. Submission is not about party’s intent, it is objectively determined by reference to the party’s actions. Entering an unqualified appearance, as opposed to one indicating a challenge to the court’s jurisdiction, can constitute submission. A: Parties hired and instructed a lawyer who made an appearance on their behalf, so they were defending on the merits and therefor submitted. NB: EE says possible loophole where an attorney goes rogue and acts w/out instructions.

CLINTON V. FORST, 1982 ONCA

F: CL action for R&E of South African judgement. This is the guy who bought a fucking Land Rover right before moving to Canada and did not pay. He was also a lawyer. He had other property in SA. Seller of vehicle started action and D was served ex juris. D files a bunch of documents in response (appearance by mail, affidavit outlining defence, statement of defence) but never challenges jurisdiction of SA court.

SA court gives judgement in favour of P; R&E action in Ontario D argues that he did not voluntarily submit but only defended to protect his land in SA from seizure

(arguing seizure of his property was a form of duress). I: Is it possible for D to appear w/out being considered to have submitted? R: If a D appears and defends case on the merits, then the appearance is voluntary. He could have appeared solely to oppose seizure or to argue no jurisdiction w/out being considered to have submitted, but he argued on merits. A: Fact that property was seized does not = free pass re voluntariness of submission. He overstepped allowable participation.

27

Page 28: cans.allardlss.comcans.allardlss.com/.../cans/Edinger_76_Fall_2017_Jasmin…  · Web viewkConflict of Laws 2017. Some General Considerations. Keep in mind that while activities being

MID-OHIO IMPORTED CAR CO. V. TRI-K INVESTMENTS LTD. (1995) BCCA

F: Breach of K action. Judgement granted in Ohio for Ohio corporation against BC company (these were the used car guys).

BC company had hired an Ohio lawyer who did three things: 1) argued no jurisdiction; 2) argued forum non conveniens; and 3) made several technical motions.

I: Did Ohio court have jurisdiction in the international sense? Did BC defendants submit by arguing forum non conveniens and/or making technical motions in Ohio? R: D for actin for R&E judgement can argue in originating action that originating court does not have jurisdiction or that originating court is not the most appropriate forum without being considered to have submitted. But if you lose on those two, stop. Withdraw from action at that point to avoid being found to have submitted. A: Defendant was not found to have submitted on the first two arguments advanced. However, was found to submit on basis of advancing the technical argument. NB: The decision was made on basis of BC Rules of Court provision that stated that foreign defendants in BC action could argue jurisdiction or forum non-conveniens without being considered to have submitted to BC Court. Court reasoned this should go both ways (otherwise foreign D’s getting an advantage not available to BC residents). But this rule is no longer in existence and no case has considered this particular issue since, so may not be good law in BC anymore.

The Common Law Rule is that if the defendant argues forum non conveniens, that will be fatal to D’s argument that they did not submit. B/c arguing forum non conveniens is asserting that court should choose to exercise discretion not to exercise jurisdiction, which indicates acceptance of jurisdiction (you can’t choose to use or not use something you don’t have).

Other Ways a D May Submit to Foreign Jurisdiction:

Submission in advance via a forum selection clause Communicating directly with foreign court Pretty much anything beyond arguing no jurisdiction could be considered submission at CL.

(iii) A Real and Substantial Connection

MORGUARD INVESTMENTS LTD. V DE SAVOYE (1990) SCC

F: Mortgage Actin in AB. D moved to BC argues no presence and therefor no submission to AB jurisdiction. I: Can an R&S Cxn b/w action and originating court create jurisdiction in the international sense wrt R7E of a foreign judgement? R: The Canadian court will R&E originating judgement (of Can court in different province/territory) if there was a R&S Cxn b/w the action and originating court. CL rules re presence and submission continue to apply. A: There was a R&S Cxn b/w action and AB. NB: Rule created in recognition of federalism—Canadian provinces ought to R&E judgements from other provinces more liberally than they R&E judgements from outside of Canada. Q then arose re whether this rule could be used for R&E of non-Canadian judgements.

BEALS V. SALDANHA (2003) SCC

28

Page 29: cans.allardlss.comcans.allardlss.com/.../cans/Edinger_76_Fall_2017_Jasmin…  · Web viewkConflict of Laws 2017. Some General Considerations. Keep in mind that while activities being

F: Saldahnas Ontario residents who bought two lots in Florida. They sold it to a developer who got himself all mixed up as what an idiot he’s been, he sues the Saldahnas in Florida. Saldahna’s consult an Ontario lawyer who says not to do anything (This guy thinks that Morguard only applies as b/w provinces, not aware that since Moses was denied leave to SCC, pple have been using Morguard to get foreign judgements R&E).

Florida court makes default judgement against the Saldanha’s and awards almost 300K in damages to Bealses. Bealses go cha-CHING and head up to Ontario for R&E.

I: Can the R&S Cxn basis for jurisdiction in the international sense be extended to R&E of non-Canadian judgements? R: Morguard rule of R&E Cxn should be extended to finding jurisdiction in international sense for non-Canadian judgments. Connection/participation must be substantial b/w Defendant and foreign court. The Cxn must be to either the action or the parties (probably could simplify by saying cxn b/w either “the action” the “proceedings” or the “dispute” per Irwin).L: Extends Morguard rule to R&E of non-Canadian judgments

Comity and prevalence of international cross-border transactions call for modernization of private international law

Real and substantial connection test requires that a significant connection exist between the cause of action and the foreign court; the connection to the foreign jurisdiction must be a substantial one

Defendant may also be brought within jurisdiction of foreign court if participated in something of significance or was actively involved in that foreign jurisdiction

A fleeting or relatively unimportant connection will not be enough to give a foreign court jurisdiction - Real and substantial connection is the overriding factor in determination of jurisdictionTraditional CL indicia of jurisdiction (submission, presence) will serve to bolster the real and substantial connection to the action or partiesA: Ds purchased land in Florida, which is a significant engagement with the foreign jurisdiction—therefore R&S Cxn b/w Florida, the subject matter, and the defendants. NB: Statement that traditional indicia of jurisdiction (submission and presence) supplement or bolster the real and substantial connection suggests that can’t just rely on submission or presence. Unclear if this would just be the case for R&E of non-Canadian judgments

Also, unclear if this real and substantial connection test for R&E of foreign judgments is identical to the real and substantial connection for jurisdiction purposes (which has been rendered quite tenuous)

Beals might suggest that there has to be a tighter connection between cause of action and originating foreign court for Canadian courts to R&E the foreign judgment (can’t trust truly foreign courts the same way can trust other Canadian courts)

SCC seemed to add some qualifications to the test in the non-Canadian foreign judgment context or to have superseded the traditional CL bases for jurisdiction but courts are just treating Beals as having extended the test and not as having superseded the traditional rules

No constitutional requirement for R&E of non-Canadian judgments (like Morguard created for R&E of Canadian judgments inter-provincially)

BRAINTECH V. KOSTIUK (1991) BCCA

F: K, BC resident, posts opinions on Braintech’s stocks, Nevada corp carrying on business in BC. Braintech has technical development activities in Texas for 3 months. Braintech commences action for defamation against K in Texas; K is not served in Texas and does not submit to jurisdiction. Braintech is successful and gets default judgment. Braintech brings action in BC for R&E of Texas judgment.I: Is there are R&S Cxn b/w the action and originating court? R: Passive posting on the internet, without more, is insufficient ground for in personam jurisdiction over a non-resident. For originating court to have jurisdiction for the purpose of R&E of foreign judgement, there must have been some purposeful activity in originating jurisdiction (according to forum’s perspective) for a R&S Cxn to be found. A: TJ erred in finding a R&S Cxn to Texas. The posting was passive activity, not purposeful commercial activity, and there was no evidence that anyone in Texas had actually accessed the posting, these combine to = no R&S Cxn b/w the action and Texas. NB: EE says the court didn’t really look at Texas rules and whether they had jurisdiction according to their own rules.

29

Page 30: cans.allardlss.comcans.allardlss.com/.../cans/Edinger_76_Fall_2017_Jasmin…  · Web viewkConflict of Laws 2017. Some General Considerations. Keep in mind that while activities being

(iv) Defences

If foreign judgment is final and conclusive and foreign court had jurisdiction in the international sense, defendant/judgment debtor can raise defenses –possible defenses:

Exclusionary rules (see above) Do not R&E foreign judgments based on penal laws, revenue laws, other public laws, or those contrary to forum

public policy Fraud in the foreign court (Beals) Breach of natural justice (Beals)

It is not a valid defence to argue that foreign court lacked jurisdiction under its own rules. It is also not open to the defendant to seek to reopen the merits of the case, either on issues of fact or law. Errors of fact or law should be raised on appeal in the foreign forum, not as defences to enforcement.

GODARD V. GRAY, 1870 QB

F: English court being asked to R&E French judgment. French judgment made by applying English law but got English law wrong (D is liable for more as result)R: Error of law by the foreign court is not a defence against forum court R&E foreign judgment.

BEALS V. SALDANHA (2003) SCC

F: Action for R&E of Florida judgment I: What CL defences are available for defendant in action for R&E of foreign judgment? R: CL defences that are available for defendant in action for R&E of foreign judgment:

Defence of fraud (fraud going to jurisdiction always, fraud going to the merits sometimes) Defence of breach of natural justice Defence of public policy

L: The CL defences are narrow in scope and are the most recognizable situations But are not exhaustive – possible that might be situation that requires creation of new defence to

enforcement of foreign judgment Defence of fraud: judgment (foreign or domestic) will not be enforced if obtained by fraud

Keep narrow to prevent re-litigation of an action Fraud going to jurisdiction can always be raised Fraud going to the merits can only be raised when the allegations are new and not the subject of prior

adjudication or when material facts not previously discoverable arise that potentially challenge the evidence that was before the foreign court

In fraud going to the merits, defendant has burden to demonstrate that facts sought to be raised could not have been discovered by exercise of due diligence before the judgment was obtained

Defence of breach of natural justice: a denial of natural justice can be the basis of challenge to a foreign judgment (aka where one of the parties was denied procedural rights considered by the forum to be fundamental or essential, such as notification of proceedings, right to present evidence, right to make submissions):

Burden on defendant to prove on civil standard (BoP) that foreign proceeding was contrary to Canadian notions of fundamental justice

Domestic court has heightened duty to protect defendant when judgment is foreign and ensure that defendant was granted fair process

Fair process is one that reasonably guarantees basic procedural safeguards such as judicial independence and fair ethical rules governing judicial system participants

Restricted to the form of the procedure, does not relate to merits of the case Look at the legal system as a whole, not at what happened in the particular case

Defence of public policy: no enforcement of foreign judgment which is contrary to Canadian concept of justice Question is whether the foreign law is contrary to forum’s view of basic morality Ex: works if foreign judgment is founded on law contrary to fundamental morality of Canadian legal

system, enforcement of judgment from court proven to be corrupt or biased

30

Page 31: cans.allardlss.comcans.allardlss.com/.../cans/Edinger_76_Fall_2017_Jasmin…  · Web viewkConflict of Laws 2017. Some General Considerations. Keep in mind that while activities being

Involves impeachment of the foreign law so should not be used lightly A: Defence of fraud: cannot be used because appellants did not provide any evidence of new and previously undiscoverable facts suggestive of fraud

Defence of breach of natural justice: no reasonable apprehension of unfairness raised by defendant (ex: were given notice)

- Defence of public policy: award does not violate our principles of morality

NON-PECUNIARY JUDGEMENTS

HUNT V. T & N PLC (1993) SCC

F: P got cancer claimed was from products containing asbestos manufactured in QB. BC court made non-pecuniary order against QB corp. for document discovery. D argues against having to comply w/ order b/c QB statute prohibiting removing certain business documents from the province. P argues the Act is unconstitutional. BC courts decline to grapple w/ this b/c they say they don’t have jurisdiction to determine constitutionality of another provinces statute. P says bullshit, federalism, morguard. Goes up to SCC. D’s say BC cout lack jurisdiction to rule on constitutionality of QB statute. I: Are the provision of the QB act a lawful excuse pursuant to BC rules of court such that D can refuse to comply with order issued under rules re disclosure? Is the statute ultra vires or constitutionally inapplicable to a judicial proceeding in another province? L/R: The rules of conflict of law do not prevent consideration of the constitutionality of the laws of another jurisdiction. The BC courts in this case did possesses at least the normal court power to consider and make findings of fact as to the constitutionality of the laws of another jurisdiction. Additionally, ability to consider constitutional issue augmented by fact that both jurisdictions are part of same Canadian federation and governed by the same constitution. La Forest does say that b/c of the far-reaching impacts of such rulings, courts should restrict themselves to hearing constitutional challenges to the legislation of OTHER provinces only where there is a real interest affected in their province.

With respect to the application of Morguard: La Forest J says: “In particular, when a corporate citizen situate in one province chooses to engage in trading and commercial activities in other provinces, the rules governing consequential litigation, specifically rules for the recognition and enforcement of judgments, should be adapted to the specific nature of the Canadian federation. And it is difficult to believe that ordinary individuals moving across Canada in the exercise of their common right of citizenship should be treated differently.”

La Forest also notes that constitutional principle in Morguard re full faith and credit being given to interprovincial governments is not within the power of the provincial legislatures to override. Provinces must respect the minimum standards of order and fairness addressed in Morguard when legislating wrt R&E of interprovincial judgements.

Court finds that Quebec statute is not consistent with Morguard principles when applied in an interprovincial context.

C: D’s have to produce, within 30 days, copies of all documents req’d per the disclosure order.

PRO SWING INC. V ELTA GOLF INC. (2006) SCC

F: Action brought against Elta in Ohio (something to do with copyright infringement). Elta submitted and defended and eventually there was a consent decree prohibiting Elta from selling any more of the clubs and requiring Elta hand over clubs to pro swing. Elta didn’t do that though and there was something like four of those clubs for sale on their website still. Po Swing finds out and goes back to court in Ohio and is successful in obtaining a contempt order. Elta still does not comply. Pro swing brings action in Ontario for R&E of Ohio contempt order (non-pecuniary order).

31

Page 32: cans.allardlss.comcans.allardlss.com/.../cans/Edinger_76_Fall_2017_Jasmin…  · Web viewkConflict of Laws 2017. Some General Considerations. Keep in mind that while activities being

I: On what basis should Canadian courts R&E non-pecuniary foreign judgements. R Non-Pecuniary foreign judgments may be R&E by Canadian courts if:

Is the Order final & conclusive and did the foreign court have jurisdiction in the international sense?

o This means via presence/submission/or R&S Cxn. Is the equitable order one suitable for enforcement?

o Do any of the traditional defences outlined n Beals prohibit R&E?

L: Extended Canadian R&E of foreign judgements to foreign non-pecuniary judgements But requires balance b/w respect for another nation’s acts and protection of enforcing jurisdictions judicial

system Analysis must be flexible b/c equity is

To R&E foreign non-pecuniary judgement, must first satisfy all rules for R&E of pecuniary judgements (final & conclusive, jurisdiction in the international sense). There are also additional considerations:

Is the order sufficiently precise and clear do that D knows what is expected from them? Is the order limited in its scope (including territorial scope)? Did the originating court retain the power to issue further orders? Is the order the least burdensome remedy for the Canadian justice system? Is the Canadian litigant exposed to unforeseen obligations? Are there any third parties affected by the order? Will the use of judicial resources be consistent with what would be allowed for domestic litigants (as far as

enforcement goes)? Remember, comity does not require courts to extend greater judicial assistance to foreign litigants than it

does to its own litigants, look to the impact of the order. Traditional defences available against R&E of pecurniary order available for D where non-pecuniary order involved. A: Court will not enforce. Final & Conclusive re an injunction can pose a bit of a problem but no comment made wrt this in this case. Jurisdiction in Int’l sense b/c of submission. But majority finds orders problematic b/c:

Contempt order is penal and Canadian courts will not enforce foreign penal judgements Territorial scope of order is not clear (is it intended to apply to Elta wherever Elta located?) Not clear that R&E of judgement is appropriate tool amongst various options

C: Primary difference b/w minority + majority concerned fact that order P sought to enforce was a contempt order. Majority classified as Penal order, as such not enforceable by Canadian courts. Minority says yeah, we won’t enforce penal law or judgement of foreign jurisdiction, but in Canada we draw a distinction b/w civil and criminal contempt, and court should enforce orders to secure compliance with a private remedy, even if granted in context of civil contempt proceedings. Remedy here was remedial and not penal, even though granted in ac contempt proceeding. But majority says no, all contempt orders have a “quasi-criminal” nature and are thus penal law.

STATUTORY REGIMES

In BC you have three choices as Foreign P with judgement against BC D. 1) Seek R&E under CL rules; 2) Seek R&E pursuant to statute; or 3) start a new action on the merits. Unclear why you would proceed with 3rd option unless absolutely necessary b/c time consuming and costly. When considering utilizing statute for R&E of foreign judgements and orders, select the appropriate statute based on the location of the originating court.

Originating court Canadian then go directly to Enforcement of Canadian Judgements and Decrees Act. If judgement or order qualifies under this act, can be registered for R&E without having to go to court.

Originating court is NOT Canadian court but is from a state that has reciprocating agreement with BC (Alaska, California, Idaho, Oregon, and California. Has agreements with Germany, Australia and the UK as well) then go to Court Order Enforcement Act .

Originating Jurisdiction NOT Canadian and NO reciprocal agreement? Use Common Law Approach Family Law Order/Judgement? Interjurisdictional Enforcement Act

32

Page 33: cans.allardlss.comcans.allardlss.com/.../cans/Edinger_76_Fall_2017_Jasmin…  · Web viewkConflict of Laws 2017. Some General Considerations. Keep in mind that while activities being

ENFORCEMENT OF CANADIAN JUDGEMENTS AND DECREES ACT , SBC 203 (ECJA)

Overview Authorizes registration of pecuniary and non-pecuniary judgements from other Canadian provinces

o Means you do not need to commence a civil proceeding for R&E, just register In force in many provinces, including BC, but not all provinces Implements judgements in blind full faith and credit Eliminates some CL defences for R&E

s. 1 Definitions

"Canadian judgment" means a judgment, decree or order made in a civil proceeding by a court of a province or territory of Canada other than British Columbia

(a) that requires a person to pay money, includingi. an order for the payment of money that is made in the exercise of a judicial function by

a tribunal of a province or territory of Canada other than British Columbia and that is enforceable as a judgment of the superior court of unlimited trial jurisdiction in that province or territory, and

ii. an order made and entered under section 741 of the Criminal Code in a court of a province or territory of Canada other than British Columbia, Note that this = pecuniary in personam judgements; implements Morguard.

(b) under which a person is required to do or not do an act or thing, or Note = non-pecuniary orders (implements Pro Swing)

(c) that declares rights, obligations or status in relation to a person or thing, Might authorize recognition of in rem judgements but not interpreted judicially.

and, subject to section 1.1, includes a domestic trade agreement award, but does not include a judgment, decree or order that

(d) is for maintenance or support, including an order enforceable under the Family Maintenance Enforcement Act,

For this you would use the Interjurisdictional Support Orders Act(e) is for the payment of money as a penalty or fine for committing an offence,

Note = Won’t enforce a penal order(f) relates to the care, control or welfare of a minor, except in the case of a Canadian civil

protection order,(g) is made by a tribunal of a province or territory of Canada other than British Columbia, whether

or not it is enforceable as an order of the superior court of unlimited trial jurisdiction of that province or territory, to the extent that it provides for relief other than the payment of money, or

Note = won’t enforce non-pecuniary tribunal order. (h) (h) relates to the granting of probate or issuing a grant of administration or the administration

of the estate of a deceased person. Won’t enforce b/c have own statutes for those issues. WESA.

s. 2 Right to register Canadian Judgement

(1) Subject to subsection (2), a Canadian judgment, whether or not the Canadian judgment is final, may be registered under this Act for the purpose of enforcement.(2) A Canadian judgment that requires a person to pay money may not be registered under this Act for the purpose of enforcement unless it is a final judgment.(3) A Canadian judgment that also contains provisions for relief that may not be enforced under this Act may be registered under this Act except in respect of those provisions.

s. 5 Time limit for registration and enforcement

1) A Canadian judgment, other than a Canadian judgment that is a domestic trade agreement award, that requires a person to pay money must not be registered or enforced under this Acta) after the time for enforcement has expired in the province or territory of Canada where

33

Page 34: cans.allardlss.comcans.allardlss.com/.../cans/Edinger_76_Fall_2017_Jasmin…  · Web viewkConflict of Laws 2017. Some General Considerations. Keep in mind that while activities being

the Canadian judgment was made, orb) later than 10 years after the date on which the Canadian judgment became enforceable in the

province or territory of Canada where it was made.1.1)sA Canadian judgment that is a domestic trade agreement award must not be registered sdffdor enforced under this Act more than 10 years after the date on which the Canadian sfsdffjudgment was made.2) Equitable doctrines and rules of law in relation to delay apply to the enforcement of a Canadian

judgment, to the extent that it provides for relief other than the payment of money.

s. 6 Application for directions (heart of the ECJA)—authorizes registration of almost any Canadian judgement

1) A party to the proceeding in which a registered Canadian judgment was made may apply to the Supreme Court for directions respecting its enforcement.

2) On an application under subsection (1), the court maya) make an order that the Canadian judgment be modified as may be required to make it

enforceable in conformity with local practice,b) make an order stipulating the procedure to be used in enforcing the Canadian judgment, orc) make an order staying or limiting the enforcement of the Canadian judgment, subject to any

terms and for any period the court considers appropriate in the circumstances, ifi) such an order could be made in respect of an order or judgment of the Supreme Court

under the applicable Rules of Court or any enactment relating to legal remedies and the enforcement of orders and judgments,

ii) the party against whom enforcement is sought has brought, or intends to bring, in the province or territory of Canada where the Canadian judgment was made, a proceeding to set aside, vary or obtain other relief in respect of the Canadian judgment,

iii) an order staying or limiting enforcement is in effect in the province or territory of Canada where the Canadian judgment was made,

iv) the Canadian judgment is contrary to public policy in British Columbia, orv) the Canadian judgment is a domestic trade agreement award and the ability to enforce that

domestic trade agreement award is, under the terms of the domestic trade agreement, suspended or otherwise ineffective.

Note: Court has power to give direction about the judgement3) Notwithstanding subsection (2), the Supreme Court must not make an order staying or limiting the

enforcement of a registered Canadian judgment solely on the grounds thata) the judge, court or tribunal that made the Canadian judgment lacked jurisdiction over the

subject matter of the proceeding that led to the Canadian judgment, or over the party against whom enforcement is sought, underi) principles of private international law, orii) the domestic law of the province or territory of Canada where the Canadian judgment was

made,b) the Supreme Court would have come to a different decision on a finding of fact or law or on an

exercise of discretion from the decision of the judge, court or tribunal that made the Canadian judgment, or

c) a defect existed in the process or proceeding leading to the Canadian judgment. Note: prohibits defendants from making certain arguments available

under CL—three things that BC recognizing court cannot consider as defence to R&E of Canadian judgment:

o Morguard and whether there was a R&S Cxn (i.e. whether court in another province properly assumed jurisdiction)

o Whether originating court made a mistake of lawo Breach of natural justice or fraud

So only defence left for D in BC under ECJA are breach of PP (unlikely to succeed). 4) An application for directions must be made under subsection (1) before any measures are taken to

enforce a registered Canadian judgment ifa) the enforceability of the Canadian judgment is, by its terms, subject to the satisfaction of a

condition, orb) the Canadian judgment was obtained without notice to the persons bound by it.

COURT ORDER ENFORCEMENT ACT RSBC 1996, PARTS 2-4

34

Page 35: cans.allardlss.comcans.allardlss.com/.../cans/Edinger_76_Fall_2017_Jasmin…  · Web viewkConflict of Laws 2017. Some General Considerations. Keep in mind that while activities being

Overview

o Used for R&E of non-Canadian judgment that is from a reciprocating jurisdictiono Do not need to commence civil process for R&E, just register ito Does not change the CL, just modifies procedure by creating registration (unlike the ECJA)o Reciprocating jurisdictions include: UK, Australia, Austria, Germany, some US states

(Washington, Idaho, Oregon, California, Colorado, Alaska).

Procedure

o Judgement creditor registers judgement from reciprocating jurisdiction ex parte so long as the time for enforcement has not expired in other state, and within 10-year limitation in BC (per s. 29)

o Creditor must notify debtor of registration w/in 30 days (per s. 34(a))o Debtor has 30 days to reply and raise an issue (e.g. lack of jurisdiction, one of the CL defences)

(per s. 34(b))o If Debtor misses 30-day deadline, judgement is enforceable

s. 29 The real Meat & Potatoes

(1) If a judgment has been given in a court in a reciprocating state, the judgment creditor may apply to have the judgment registered in the Supreme Court unless

(a) the time for enforcement has expired in the reciprocating state, or(b) 10 years have expired after the date the judgment became enforceable in the reciprocating state.

(1.1) On application under subsection (1), the Supreme Court may order that the judgment be registered.(2) An order for registration under this Part may be made without notice to any person in any case in which

(a) the judgment debtor(i) was personally served with process in the original action, or(ii) although not personally served, appeared or defended, or attorned or otherwise submitted to the jurisdiction of the original court, and

(b) under the law in force in the state where the judgment was made,(i) the time in which an appeal may be made against the judgment has expired and no appeal is pending, or(ii) an appeal has been made and has been disposed of.

(3) In a case to which subsection (2) applies, the application must be accompanied by a certificate issued from the original court and under its seal and signed by a judge or the clerk of that court.(4) The certificate must be in the form set out in Schedule 2, or to the same effect, and must set out the particulars as to the matters mentioned in it.(5) In a case to which subsection (2) does not apply, notice of the application for the order as is required by the rules or as the judge considers sufficient must be given to the judgment debtor.(6) An order for registration must not be made if the court to which the application for registration is made is satisfied that

(a) the original court acted either(i) without jurisdiction under the conflict of laws rules of the court to which application is made, or(ii) without authority, under the law in force in the state where the judgment was made, to adjudicate concerning the cause of action or subject matter that resulted in the judgment or concerning the person of the judgment debtor,

(b) the judgment debtor, being a person who was neither carrying on business nor ordinarily resident in the state of the original court, did not voluntarily appear or otherwise submit during the proceedings to the jurisdiction of that court,(c) the judgment debtor, being the defendant in the proceedings, was not duly served with the process of the original court and did not appear, even though he or she was ordinarily resident or was carrying on business in the state of that court or had agreed to submit to the jurisdiction of that court,

35

Page 36: cans.allardlss.comcans.allardlss.com/.../cans/Edinger_76_Fall_2017_Jasmin…  · Web viewkConflict of Laws 2017. Some General Considerations. Keep in mind that while activities being

(d) the judgment was obtained by fraud,(e) an appeal is pending or the time in which an appeal may be taken has not expired,(f) the judgment was for a cause of action that for reasons of public policy or for some similar reason would not have been entertained by the registering court, or(g) the judgment debtor would have a good defence if an action were brought on the judgment.

(7) Registration may be effected by filing the order and an exemplification or certified copy of the judgment with the registrar of the court in which the order was made, and the judgment must be entered as a judgment of that court.(8) If a judgment provides for the payment of money and also contains provisions for other matters, the judgment may only be registered under this Part for the payment of money.

IN PERSONAM CLASS ACTIONS : JURISDICTION AND R&E

Two models of class actions1. Opt-out Model

o Court declares a class (could even be national or international class) andyou have to opt out of class.2. Opt-in Model

o If you want to join class action, you have to opt ino This is the model in BC

Conflicts of Law Issues Arise in Class Actions:

o Questions of jurisdiction simpliciter/territorial competenceo Questions of appropriateness/forum convenienso If there is already a class action underway in another jurisdiction, should one be stayed in favour of the other,

should they both continue? o Questions about R&E of class action judgement.

JURISDICTION ISSUES IN CLASS ACTIONS (JURISDICTION SIMPLICITER/TERRITORIAL COMPETENCE AND FORUM NON-CONVENIENS)

HARRINGTON V DOW CORNING CORP (2000) BCCAF: Class action brought in BC (opt in province) against Dow for faulty breast implants. Both resident class, and nonresident classes (Some non-residents wanted to opt in to BC class action) wanted to opt in. If you opt in, you submit to the jurisdiction of court. Dow objected to BC court taking jurisdiction over these non-residents (Dow’s strategy was divide and conquer, probably gambling that not all actions would get off the ground).

Dow argued that class plaintiffs should be limited to those whose claims had R&S Cxn with BC (if you didn’t get your implants in BC, no R&S Cxn).

I: For a class action, does there have to be a R&S xn b/w forum and each plaintiff in the plaintiff class? In other words, can forum court assume jurisdiction over a non-resident class? R: A common issue between the non-resident and resident class is a sufficient R&S Cxn to allow non-residents to opt in and forum court to take jurisdiction. L: Where traditional rules (presence, submission) are not adequate to ensure fairness and order, then other considerations become relevant, including the nature of subject matter of the action, and the existence of a certified class proceeding. A certified class action with a common issue may provide R&S Cxn b/w province and subject matter of the claim that a non-resident seeks to have resolved in the same class proceeding. A: BCSC properly assumed jurisdiction over all claims by purchasers resident in BC and has certified an issue common to all purchasers for resolution in the class proceeding.

Excluding plaintiffs who do not reside in BC from this action b/c they have no used the product in BC would

36

Page 37: cans.allardlss.comcans.allardlss.com/.../cans/Edinger_76_Fall_2017_Jasmin…  · Web viewkConflict of Laws 2017. Some General Considerations. Keep in mind that while activities being

contradict the principles of order and fairness that underlie jurisdictional rules.

WARD V. CANADA (2007) MBCAF: Class action commenced by MB resident against Canada for herbicide spraying on military base in NB (Ps were soldiers on base at time of spraying and allegedly suffered major health consequences). Action at early stage and appeal concerned jurisdiction of Manitoba court (opt out province). Court noted that Crown is resident in every province, and jurisdiction over Manitoba resident plaintiff and defendant, but that it was very likely that nonresidents would be included in the class of plaintiffs if/when class identified and certified. Crown argued that only NB could properly take jurisdiction, and in the alternative, if MB did have jurisdiction, should exercise discretion and defer to courts of NB. I: Should there be a new or modified approach to jurisdiction in a proposed class action, in circumstances where, if the action was an individual action, the court would clearly have jurisdiction? R: Traditional rules of presence and submission still apply to give court jurisdiction in class actions, crown present in every province, so MB courts have jurisdiction over crown. L: 1. Jurisdiction SimpliciterTraditional CL rules of presence and submission still apply to give the court jurisdiction in the case of a class action. The crown’s presence in MB satisfies the traditional test outlined in Moran v. Pyle.

Potential inclusion of non-residents in class of plaintiff should not deprive a court of the jurisdiction simpliciter it would otherwise have (per Dow Corning C=orp.).

Here the crown has not demonstrated (as yet) why the traditional, presence based approach to establishing jurisdiction would result n other than fairness and order. R&S Cxn test does not override traditional test, and no reason to wade into R&S Cxn test here as traditional test applies (presence).

Court finds that prospective certification as class action does not deprive court of jurisdiction simpliciter. This is b/c class actions are a procedural vehicle and does not change the legal rules relating to subject matter jurisdiction. Class action procedure cannot have the effect of conferring jurisdiction over a group of cases that would otherwise fall into subject matter jurisdiction of another court or tribunal

The prospective addition of non-resident plaintiffs to the class would only become relevant at the certification or in the forum non conveniens analysis

2. Forum Non-ConveniensGiven court has JS, should it decline to exercise it? Only if there is a forum that is more appropriate or convenient for the judicial resolution of the claim. This is a high standard per Amchem: existence of more appropriate forum must be clearly established to displace the forum elected by P.

For forum non conveniens in a class action, consider the usual factors + whether there is a parallel action. 3. The court has discretion to reconsider jurisdiction or forum non-conveniens later in the proceeding for certification (ex if it receives more information re plaintiff class). A: Jurisdiction simpliciter established at this stage. B/C (1) the presence here of the plaintiff, (2) his as yet unchallenged assertion of damage sustained here, (3) the presence here of the Crown, (4) the clear juridical advantage to the plaintiff of prosecuting a class action in this province, and (5) no apparent significant juridical disadvantage to the Crown, there was sound reason for the judge to have exercised his discretion in favour of this province as forum conveniens. Finally, there will be future opportunities for MB court to take into account events that might occur in the courts of other provinces and, if appropriate, make an order that it is no longer forum conveniens.

KAYNES V. BP PLC (2014) ONCAF: Class action brought in Ontario against BP by Shareholders for misrepresentation (based on ON Securities Act). Proposed class is all residents of Canada who acquired BP securities in date range. ONCA locates tort of misrepresentation in Ontario. I: Does Ontario have jurisdiction over the claim of the P and proposed class members? If so, should it exercise that jurisdiction. R: To Establish jurisdiction of court in a class action under an R&S connection, one of the four presumptive connecting factors from Van Breda can be used. These are: D is domicile or resident in province, D carries on business in the province, tort committed in province, K connected w/ dispute made in the province. A: Tort occurred in Ontario therefor there is a cxn b/w action and the jurisdiction and Ontario has jurisdiction simpliciter. BUT Ontario forum non-conveniens b/c BP establishes existence of clearly more appropriate forum in US, therefore Ontario action stayed b/c another class action in US. Commentary: Later on Ps ask ON to reconsider and court lifts stay and class action now proceeding (evidence that US court where parallel action was eventually refused to proceed).

RECOGNITION AND ENFORCEMENT OF CLASS ACTIONS

37

Page 38: cans.allardlss.comcans.allardlss.com/.../cans/Edinger_76_Fall_2017_Jasmin…  · Web viewkConflict of Laws 2017. Some General Considerations. Keep in mind that while activities being

Defendants in class actions can argue for R&E of foreign class action judgment as shield against new class action

- Defendants want to enforce the judgment as widely as possible in order to preclude further litigation against them

o Argue that there is already a judgment against us; plaintiff is member of the class in the other action and is therefore bound by that result

- Therefore, question arises of whether the originating foreign court (that issued the class action judgment) had jurisdiction over the plaintiff that now wants to start a new class action in another jurisdiction

o Versus traditional non-class action suit, where no question over jurisdiction of foreign court to bind the plaintiff because the plaintiff initiated the proceeding and therefore attorned to the foreign court’s jurisdiction

- Note: any defendant can argue this but just doesn’t come up much in non-class action private cases

CURRIE V. MACDONALD’S RESTAURANTS, 2005 ONCAF:

Class action in Illinois against McDonalds for consumer fraud and unjust enrichment Illinois court declared international class that extended to Canadians For class action, court has to give proper notice and give plaintiffs an opportunity to opt out Notices for this class action were placed in Macleans magazine Currie wants to start own class action; argues that he didn’t see the notice and couldn’t opt out

I: Is a plaintiff bound by a class action settlement if the plaintiff failed to opt out? What are the notice requirements for a class action?

R: - Criteria for R&E of foreign class action judgment

o Real and substantial connection between the cause of action and the foreign jurisdiction (court must have had jurisdiction in the international sense)

o Rights of the non-resident class members must have been adequately represented

o Non-residents class members must have been accorded procedural fairness: includes adequate (and clear) notice of the class action and opportunity to opt out

- If doesn’t meet these three requirements, then won’t be R&E against new plaintiffsL:

R&E rules should take into account certain unique features of class action proceedings Need to ensure that foreign court had proper basis for assertion of jurisdiction over forum

resident plaintiffs Wrt real and substantial connection for jurisdiction, consider the perspective of non-resident

plaintiff who has done nothing to invoke or submit to the jurisdiction of the foreign court If the three criteria (above) are met, may be appropriate to attach jurisdictional consequences to

unnamed plaintiff’s failure to opt out (failure to opt out = form of passive attornment sufficient to support jurisdiction of foreign court)

But not determined by a rigid test May be easier to justify assumption of jurisdiction in interprovincial case than in international

case A:

(1) Illinois court did have jurisdiction in the international sense (wrong occurred in US, McDonalds head office is in Illinois)

(2) The non-residents were adequately represented(3) But procedural fairness not satisfied – notice and opportunity was not adequate (ad was wall

to wall legalese, dissemination inadequate (how many people read Macleans?))COMMENTARYEE says third requirement amounts to defence of natural justice

38

Page 39: cans.allardlss.comcans.allardlss.com/.../cans/Edinger_76_Fall_2017_Jasmin…  · Web viewkConflict of Laws 2017. Some General Considerations. Keep in mind that while activities being

MEEKING V. CASH STORE AND INSTALOANS, 2013 MBCA 81 P. 479 of Case bookF:

Series of class actions brought against pay day loan stores Ontario class action settled, Ontario class had extended to everyone in Canada except residents of

BC/AB (b/c those provinces had concurrent actions) Meeking, MB resident, failed to opt out of ON action and wants to start class action in MB Cash Store applies to MB court for order R&E Ontario settlement (argues that Meeking is bound by ON

order)I: Does the common issue standard work as a real and substantial connection for the purposes of R&E a foreign class action judgment?R: If foreign court had territorial jurisdiction over defendant and representative plaintiff + there is a common issue between the claim of the representative plaintiff and the claim of the non-resident plaintiff(s), then there is a presumptive connection that gives the foreign court jurisdiction over the non-resident plaintiff(s)L: Apply Van Breda principles for recognition of new presumptive connecting factor (1) D is domicile/resident in province, 2) D carries on business in the province, 3) tort committed in province, 4) K connected to dispute was made in the province).

Common issues factor is similar to recognized presumptive connecting factors in that it deals with the subject matter of the wrongdoing (wrongs committed are the same across the country)

Van Breda stated that connections with the province need not be the strongest ones possible in order to fulfill the real and substantial connection test

Recognizing common issue as presumptive connecting factor when court has territorial jurisdiction over defendant and rep plaintiff is consistent with principles of order, comity, and fairness

Therefore, when court has territorial jurisdiction over both the defendant and representative plaintiff in a class action proceeding, common issues between the claim of the rep plaintiff and that of non-resident plaintiffs is a presumptive connection factor sufficient to give the court jurisdiction over non-resident plaintiffs

This is presumptive connecting factor and can be rebutted Wrt R&E of foreign judgment (once determine that foreign court had jurisdiction), see CurrieA:

Ontario court had jurisdiction over the defendant (carried on business in Ontario) and representative plaintiff (resident, submitted); and there is a common issue

Therefore, Ontario court properly assumed jurisdiction over MB plaintiffC:

SCC granted leave to appeal but hearing adjourned because Cash Stores commenced proceedings under Companies Creditors Arrangement Act

o People wanted to know whether common issues constitute a R&SC for jurisdiction and as basis for R&E

39

Page 40: cans.allardlss.comcans.allardlss.com/.../cans/Edinger_76_Fall_2017_Jasmin…  · Web viewkConflict of Laws 2017. Some General Considerations. Keep in mind that while activities being

IN REM ACTIONS: JURISDITCION AND R&E

In Rem actions involve rights that are, in theory, good against the world. They have special jurisdictional and R&E rules. Type of in rem actions

Title to propertyo Property in question must be classified as movable property or immovable property

Note: this does not align perfectly with personal property and real property Interests in property can include in vivos transfer, testamentary transfers, trusts for sale,

sometimes even shares Questions of status (married, divorced, not married, not divorced) Actions against the ship

CLASSIFICATION OF PROPERTY

Classification of property (as movable or immovable) is relevant for characterization, jurisdiction, and R&E of foreign judgements and determination of the applicable law. The distinction is particularly important in succession b/c general CoL rule that succession of moveable property is governed by law of deceased’s domicile and succession of immovable property is governed by the law of the situs of the land.

Immovables are situated where land is situated and chattels are situated where they are physically located. Choses in action are generally situated n the country where they are properly recoverable or can be enforced (Dicey Rules).

The Forum determines the situs of property. Then you must classify the property. Question of whether property is immovable or movable must be determined by law of place the property is situated (Hogg). Then, depending on whether moveable or immoveable, law of forum or law of domicile applies to disputes concerning that property interest.

40

Page 41: cans.allardlss.comcans.allardlss.com/.../cans/Edinger_76_Fall_2017_Jasmin…  · Web viewkConflict of Laws 2017. Some General Considerations. Keep in mind that while activities being

HOGG V. PROVINCIAL TAX COMMISSIONER (1914) SKCAF:

SK tax commissioner wants to impose tax on estate of deceased who died domicile in SK Will disposed of estate which included mortgages in land in BC Under SK tax act, SK can only impose tax if property in will devolve under SK law – this depends

on whether mortgages are movables or immovable o If movables, they will devolve upon beneficiaries under law of SK o If immovable, they will devolve upon beneficiaries under the law of BC

I: Can Sask impose on estate of individual who died domicile in Sask where a portion of the property is mortgages on land in BC? Depends on whether mortgages classified as moveable or immoveable.

How is the question of whether property is movable or immovable determined? Once determine if property is movable or immovable, what law governs?

R: Question of whether property is movable or immovable must be determined by the law of the place where the property is situated

Question of fact that must be determined by expert evidence (legal experts from that place) Interests in land are located where the land is located

Law of the situs (where the property is located) governs succession to immovable property. Forum accepts absolutely whatever characterization is placed by the situs of the immovable. Law of the domicile governs succession to movable property

Look to the law of the place where the property is situated and determine how that legal system would characterize the asset (movable or immovable)

A: A mortgage is an interest in land and the land is located in BC Therefore, need expert evidence from BC lawyers on whether law of BC considers mortgages to

be movable or immovable property BC considers mortgages to be immovable property Therefore, mortgages devolve upon beneficiaries by law of BC and no tax can be imposed by SK

C: How a legal system classifies property as movable or immovable may be based on arbitrary rule

o Note that approach to characterization re interests in land is different than how it generally works, wrt interest in land forum accepts absolutely the characterization that law of situs would apply to that interest.

o You only need expert classification, relying on this court can decide whether interest devolves under law of forum or situs. Expert must be from situs.

JURISDICTION IN IN REM ACTIONS

Under CJPTA, B.C. will have jurisdiction over property if it is located in B.C. (regardless of whether property is movable or immovable)

- Problem for courts arises when property is classified as immovable property and is located outside of the court’s jurisdiction

o Rule: an English court does not have jurisdiction over immovable property (ex: title to or action in trespass against) if that immovable property is located outside of forum

Rule from Dicey

- Subject to the exception below, a court has no jurisdiction to entertain proceedings for (1) determination of title to or right to possession of immovable property situated outside of England or (2) the recovery of damages for trespass to such immovable (absolute Mocambique rule)

- Exception: the court has jurisdiction to entertain an action against a person who is in England respecting an immovable located outside of England on the ground of either

o (1) The claim is based on a contract or equity between the parties Contract about immovable between the parties, or Equity between the parties that produces an equitable order (ex: anti-suit injunction)

o Or, (2) The question has to be decided for purpose of administration of an estate or trust and the property consists of immovable or movable property in England and immovable property outside of England

41

Page 42: cans.allardlss.comcans.allardlss.com/.../cans/Edinger_76_Fall_2017_Jasmin…  · Web viewkConflict of Laws 2017. Some General Considerations. Keep in mind that while activities being

BRITISH SOUTH AFRICA CO. V COMPANHIA DE MOCAMBIQUE (1893) HLF: Dispute in England of allegation of trespass on mines and minerals in South Africa. Mines and minerals are immovable property. I: Does the court have jurisdiction to settle a dispute over immovable property outside the jurisdiction?R: Court does not have jurisdiction over cases that involve trespass to foreign immovable property Rationale for rule

- Risk of inconsistent decisions - Lack of control (court cannot control what happens to that property (ex: cannot make order to

transfer title)

This has been affirmed in Canada in Tezcan v. Tezcan where McLachlin JA expressed the rule as follows:

The general rule is that courts of a country have no jurisdiction to adjudicate on the right and title to lands not situate w/in its borders. Only the courts of the jurisdiction in which the lands are situate may adjudicate on the rights and title to such land.

HESPERIDES HOTELS LTD. V. MUFTIZADE (1970) HLF: Greek owners of hotel in Cyprus (immovable property) which has been occupied by Turkish operators following invasion, find out (English?) agency booking people into the hotel. Owners, who are in England, commence action in England for tort which could be located in England (conspiracy to trespass). Attempt to avoid Mocambique rule (by not calling it action in trespass or over title). Defendants argued that English court did not have jurisdiction based on Mocambique.I: Does Mocambique rule still apply?R: Mocambique rule still applies L: 4 reasons to not overrule Mocambique rule

(1) Lots of CL countries have adopted the Mocambique rule (2) This is matter of delicacy which should be left to legislature (3) Revision might produce forum shopping and we aren’t ready for that (4) Nothing has changed since 1893 – we still can’t control what happens with property in foreign jurisdiction and might have inconsistent decisions

A: Plaintiff’s argument is just trying to do indirectly what cannot be done directly; fails

A CANADIAN QUALIFICATION OF MOCAMBIQUE RULE?

GODLEY V. COLES (1988) ONT DCF: Plaintiff and defendant are both Ontario residents. Both had foreign property in condo in Florida; P lived directly below the D. D’s toilet leaked, water ran into P’s condo and cause lots of damage (to movable and immovable property)/ D argued that Ontario court did not have jurisdiction based on Mocambique (and was not the most appropriate forum)I: If the primary damage to property is to movable property, does some damage to immovable property raise the application of the Mocambique principle so that plaintiffs are precluded from suing in a court which is not the court where the immovable property is located? Does Mocambique rule apply in cases where title is not in dispute?R: If a large portion of the damage caused by the defendant is damage to movable property, then the presence of some damage to immovable property should not disentitle plaintiffs from bringing an action in the province the two parties are resident in. (court questioned whether Mocambique rule applicable in every situation where % in foreign land involved but primary qualities of action related to action in personam).

- Note: court can assume jurisdiction over an action in which the plaintiff seeks to recover damages caused by negligence to movables in another jurisdiction.

A: Large portion of damage was to the contents of the condo (movables), therefore can bring action in Ontario even though there was some damage to immovable property. Title is not in disputeC: This might be Canadian qualification of the Mocambique rule but not sure since case has not been used since.

42

Page 43: cans.allardlss.comcans.allardlss.com/.../cans/Edinger_76_Fall_2017_Jasmin…  · Web viewkConflict of Laws 2017. Some General Considerations. Keep in mind that while activities being

EXCEPTION TO MOCAMBIQUE RULE BASED ON K OR EQUITY B/W THE PARTIES

WARD V. COFFIN (1972) NBSC F: K b/w P and D for sale of land (immovable property) in QB, valid contract made in NB. D doesn’t follow through w transfer of land, served in NB (court has jurisdiction over D). Plaintiff sought specific performance or, in alternative, damages for breach of K. I: Does the court have jurisdiction over this dispute related to foreign immovable property?R: The court has jurisdiction over a dispute about foreign immovable property if the claim is based on a contract or equity (exceptions to Mocambique rule) A: Contract is about transfer of title to foreign immovable but cause of action in based on contract, therefore court has jurisdiction

RECOGNITION AND ENFORCEMENT OF IN REM ACTIONS

Converse of the foreign immovable rule is that courts of a foreign country have no jurisdiction to adjudicate title to, or the right to possession of, land outside that foreign country.

DUKE V. ANDLER, 1932 SCC F:

Contract made in California between Cali vendor and purchaser for sale of immovable property in BC Purchaser gets hands on title deeds without paying for property Vendor commence action in contract against purchaser in Cali (wants equitable order for specific

performance) Cali court took jurisdiction and ordered D to re-convey property to vendor; they don’t Cali court ordered county clerk to re-convey title to vendor Clerk went to BC to re-register title in vendor’s name; Registrar of land titles declined to do so Plaintiffs sued in BC to enforce judgment/order for specific performance of Cali court

I: Should the court R&E the foreign judgment that purports to decide who has title to property in the court’s jurisdiction (via an equitable order)?R: Canadian CL courts will not R&E foreign judgments dealing with local immovable property

Based on Dicey rule that courts have a foreign country had no jurisdiction to adjudicate upon the title or right to the possession of any immovable not situated in that country; judgments of foreign courts purporting to deal with the title and with rights to lands in another country can only be enforced by proceedings in personam

A: Cali court had jurisdiction in the contract action but cannot give effect to order that purports to give Cali court official authority to transfer the property; therefore no recognition COMMENTARY

Duke sounds absolute but there are likely exceptions to it now: Pro Swing, ECJA Application of Pro Swing to this situation

o Now, if you were CA plaintiff, you would rely on Pro Swing: ask BC courts to R&E equitable order of foreign court

o BC court would have to consider in light of Pro Swing and factors set out o So result of Duke might be different today o Note: even though Cali is reciprocating state, could not use COEA because order for specific

performance because CEOA only applies to pecuniary judgments Application of ECJA to this situation

o If Cali was a Canadian province, might be covered under ECJAo S. 1(c) of ECJA defines a Canadian judgment as one that declares rights, obligations, or status

in relation to a person or thing – land is a thing o Could argue that s. 1(c) authorizes BC court to recognize equitable order originating in

another Canadian court, even if the order affects land in BC Issue is unlikely to arise often because most CL courts follow Mocambique

o So, unlikely that foreign CL court will take jurisdiction to determine title or right to possession of BC land because Mocambique rule pretty entrenched

o But Mocambique rule can be overruled by statute (statute might say that court can take jurisdiction over immovable property outside the jurisdiction)

43

Page 44: cans.allardlss.comcans.allardlss.com/.../cans/Edinger_76_Fall_2017_Jasmin…  · Web viewkConflict of Laws 2017. Some General Considerations. Keep in mind that while activities being

CHOICE OF LAW

Choice of law rules = rules to decide what law to apply to the merits of the action (once court takes jurisdiction)

Approach in Canada: standard/classical/traditional jurisdiction selecting rules Choice of law rules typically formulated in terms of an issue and a connecting factor that points to some legal

system o Ex of connecting factors: place of celebration of marriage, domicile, place of commission of the tort

Don’t have to argue for application of any foreign law if you don’t want. Court will only consider Choice of Law if one party raises it as an issue. Reasons why CoL might not be raised:

o Deliberate – counsel looked at other legal systems and would not benefit your client/same result o Accidental – don’t know there is an option to apply foreign law

If want to argue for application of foreign law, must plea and prove foreign law to satisfaction of the court o Have to get foreign expert (foreign law is considered a matter of fact in the action)o Must cite relevant choice of law rule that governs the issues in the action to get to the foreign law!

Choice of law rule only comes into play if a party to the legal dispute proves that (1) there is an applicable choice of law rule, (2) the choice of law rule does embrace, by the category of issues to which it refers, the foreign legal rule the party relies on (are they the same category of rule), (3) that the choice of law rule, via its connecting factor, does point to the foreign country in question, and (4) that the choice of law rule should be applied so as to make the foreign legal rule operate to establish the party’s claim or right

Step 4 may not happen because e.g.: foreign rule of law barred from application because it is a penal law, tax law, or against forum public policy; ambiguities in the rule (renvoi)

If neither party raises the question of choice of law or a party that does raise the question fails to prove what the relevant foreign legal rule is or how it applies, then the court applies its own law

Process of applying a choice of law rule

(1) Characterize the issue (is it within the category of issues to which the rule applies)(2) Follow the connecting factor to a particular legal system (3) Apply the law that you find in the country to which the connecting factor has led you

Dicey Rule 1: Defining the law of a country

(1) When applied to English law, the law of the country means the domestic law of England(2) When applied to any foreign country, the law of the country usually means the domestic law of that country

RENVOI

When the Conflict of Laws rule of the forum refers a jural matter to a foreign law for decision, is the reference to the corresponding rule of the Conflict of Laws of that foreign law, or is the reference to the purely internal rules of law of the foreign system; i.e., to the totality of the foreign law minus its Conflict-of-Laws rules (per Professor Schreiber).

Dicey’s Rule 1 says the forum’s conflicts choice of law rule usually points to the domestic law of the lex causae (the place that the conflicts rule points you to). BUT sometimes the forum’s conflicts choice of law rule points to the conflicts law of the lex causae.

You have to consider whether you will argue:

- No renvoi (confine yourself to proving substantive law of foreign court)- Partial renvoi- Or Total/Double Renvoi

Partial Renvoi is when the court of the forum has a choice to apply the foreign choice of law rules, accept the remission to its law by the foreign law and apply the law which it would have applied had the case been entirely

44

Page 45: cans.allardlss.comcans.allardlss.com/.../cans/Edinger_76_Fall_2017_Jasmin…  · Web viewkConflict of Laws 2017. Some General Considerations. Keep in mind that while activities being

domestic to the forum, or in the case of transmission, the domestic law of the third country. This requires proof of the choice of law rules of the foreign country but not of the foreign rules about renvoi. This is called single renvoi.

- Use this when you don’t get the right result by applying domestic law of the lex causae- Must prove the choice of law rules of the foreign jurisdiction

Total or Double Renvoi is when the court of the forum may resolve the issue in the same manner as a court of the legal system selected by its choice of law rules might resolve it had the foreign court exercised jurisdiction in the same case on the same facts. This method requires proof not only of the choice of law rules of the foreign country but also the foreign rules about renvoi. This is called double renvoi.

- Here you are asking what would the foreign legal system do with the issue? o Would the foreign court look to conflicts rules of the other jurisdiction? o Must have an expert in the foreign legal system/the lex causae tell the court how a court of the lex

causae would solve this problem. o Parties are bound by what the expert says/forum court will choose which of the battling experts they

believe. o Possible options: apply domestic law of lex causae, apply conflicts rule of lex causae; apply renvoi, or

total renvoi- If lex causae expert says that lex causae would apply total renvoi, this points back to doing what the forum

court would do; the forum court says we do whatever the lex causae would do—on and on forever!o But this has never actually happened, rare for courts to actually apply total renvoi.

- To avoid lex causae, ask expert “what is your domestic law” and “what is your conflicts law” (rather than “how would your court solve this”)

- Must prove choice of law rules of foreign court and foreign court’s renvoi rules.

In Tezcan v. Tezcan Judge Cummings gives a great summary of Renvoi:

99 (i) No renvoi — reference to the law of the foreign jurisdiction is taken to be only the substantive domestic law of that system. It is not to include any of the conflicts rules of that jurisdiction. Only the substantive law of that jurisdiction need be proven as a matter of fact.

100 (ii) Partial renvoi — reference to the law of the foreign jurisdiction is taken to include not only its substantive domestic law, but also the choice of law rules of that system. The court does not ask simply what the domestic law of the foreign jurisdiction is, but also what law the other jurisdiction would look to. The court then applies the law of a third jurisdiction (transmission), or its own law (remission) if that is what the choice of law rules of the foreign jurisdiction dictate. The choice of law rules of the other jurisdiction must be proven as a matter of fact.

101 (iii) Total renvoi — The forum court chooses which system of law to apply based on its own choice of law rules. It must then must decide the case exactly as if it were the court of the jurisdiction chosen. Therefore, it must ask itself not only which law the foreign court would apply having regard to its conflicts rules, but also whether the foreign court would look to the conflicts rules of other jurisdictions (i.e., would it apply renvoi). This requires proof of the foreign courts' choice of law rules as well as its rules regarding renvoi.

Ares of law that renvoi is used in: Family (courts want to find marriage valid), succession (courts want to uphold wills), immovable property (deference to what foreign court would do). There is no renvoi in contracts, many statutes exclude renvoi.

NIELSON V. OVERSEAS PROJECTS CORPORATION OF VICTORIA LTD ., 2005 HCA (AUSTRALIA)F: N’s husband working for OPC (Australian company) in China, which provides couple housing; N falls on OPC property in China. N commences action against OPC in Australia. Australia’s choice of law rule for tort: law governing the action in tort is the law of the place where the tort occurred (lex loci delicti); limitation period that applies is limitation period of law where tort occurred (Remember Tolofson, limitation part of substance not procedure). Limitation period in Chinese law was 1 year and 1 year had passed – bad for N. Limitaton period in Australia longer, and she’s golden if Aust. Limitation applies.I: Should the court apply Chinese law or Australian law? If former, does Renvoi apply in any capacity?R: Renvoi is still alive! L: Lex loci delicti refers to the whole law of the foreign jurisdiction (including conflicts law)A:

45

Page 46: cans.allardlss.comcans.allardlss.com/.../cans/Edinger_76_Fall_2017_Jasmin…  · Web viewkConflict of Laws 2017. Some General Considerations. Keep in mind that while activities being

Distinction b/w domestic law of lex causae and it’s conflict rules is arbitrary. Where forums CoL rule depends on connecting factor other than place, law is the whole of the law of that place (aka in tort action, when we look to lex loci delicti—we look whole law).

Because lex loci delicti refers to the whole law, this Chinese article should be considered Chinese article wrt compensation for damages states that the law of the place in which the

infringement occurred shall be applied excepto Article has qualification: if both parties are nationals of the same country, the law of

their own country may also be applied This qualification means that this rule is a conflicts rule. Apply renvoi – Chinese conflicts rule means remission back to Australian domestic law (no evidence

that it was intended to have effect of pointing to Australia choice of law conflicts rule)o Australia limitation period has not expired, so N can proceed.

C: Has not yet been followed by Canadian court but might be…

MARRIAGE

Validity of marriage often arises as subsidiary question in connection to main question (ex: succession)

Marriage must be both formally valid and essentially valid! Public policy often plays significant role in validity of marriage determinations (so can argue public policy

depending on the answer you want)

Characterization of defects of marriages

Defects that go to formal validity: notices, number of witnesses, registration (was the marriage required to be registered, was it properly registered), civil and religious marriages (are both required), proxy marriages, online marriages, parental consent (note: civil law courts consider this to be question of essential validity)

Defects that go to essential validity: age (is there a minimum age), consanguinity (blood relationship restrictions), affinity (relationships by marriage), single status, consent (can be vitiated by fraud, duress, mistake (as to nature of ceremony, identity of other party, as to the attributes of the other party), mental illness, mental reservations), impotence, numbers (how many people can you be married to at once), gender.

BROOK V. BROOK, 1891 HLF:

W married C (3 children); C dies W marries E, C’s sister (2 children); but English statute prohibited marriage to sister in-law or brother in-

law so W and E went to Denmark to get married where this was allowed W and E both die; will leaves property to all 5 children; 1 child of W and E dies AG argues that second marriage is void (therefore property of deceased child should pass to Crown)

I: How is the validity of a marriage determined?R:

2 rules to determine validity of the marriage Formal validity: governed by the law of the place where the ceremony occurred Essential validity: governed by the dual domicile rule (each party must have capacity by their

domicile to marry the other party) or by the law of the intended matrimonial home (not alternative rules but either/or rule)

Marriage must be both formally valid and essentially validA:

Formal validity: governed by Denmark law – marriage is formally valid Essential validity: both W and E domicile in England so apply the law of England – not essentially valid

(they never intended to live in Denmark so no option to apply this test instead) B/C not essentially valid, marriage is void

C:

HL did not like W and E avoiding English law so created the essential validity rule

I. Formal Validity

Rule: to determine if marriage is formally valid, apply the domestic law of the place where the ceremony occurred (Brook v. Brook). If both parties complied with law of the place where ceremony celebrated, they have a formally valid marriage.

o May also be formally valid if it complies with one of alternative: Partial Renvoi (per Taczanowska)—A marriage will be formally valid if it complies with the

formalities imposed by the domestic law of the place of celebration or with those imposed by the domestic law identified by the choice of law rules of the law of the place of celebration.

46

Page 47: cans.allardlss.comcans.allardlss.com/.../cans/Edinger_76_Fall_2017_Jasmin…  · Web viewkConflict of Laws 2017. Some General Considerations. Keep in mind that while activities being

Common law marriage: Deals with req’ for marriage prior to legislation. Agreement to marry followed by period of cohabitation with reputation of being husband and wife. Applied where there is no possibility of complying with local law (e.g. refugee camps).

TACZANOWSKA V. TACZANOWSKI, ENG CAF: Two Polish nationals get married in Italy (marriage performed in Church by priest). Priest forgot to read

out certain provisions and failed to register the marriage (alleged defects of formal validity).I: Can this marriage be found to be formally valid?R:

Choice of law rule for formal validity of marriage: marriage is formally valid if complied with law of the place where the marriage was celebrated

o First, look at domestic law of place where the marriage was celebratedo Can also use the choice of law rule of the place where the marriage was celebrated (partial

renvoi)- Marriage can also be formally valid if there was a valid common law marriage

A:

If look at Italian domestic law – failed to comply with it so marriage not formally valid via Italian domestic lawLook at the Italian choice of law rule – connecting factor for Italian conflict law is nationality

- Nationality of parties is Polish so transmission to Polish law- Did not meet requirements of Polish law so marriage not valid this way either

But does meet requirements of common law marriage therefore formally valid!

II. Essential Validity

Concerns the legal capacity of the parties to marry. Rule: to determine if marriage is essentially valid, apply either the dual domicile rule or the intended matrimonial

home rule (Brook v. Brook)o Not yet decided which of these rules is the proper choice of law rule for essential validity of marriage

Although dual domicile rule is the one applied most frequently in the case law May have to justify application of intended matrimonial home rule based on facts because dual

domicile rule seems to be the standard o Ask the court to apply the test that is more favourable to your client

SANGHA V. MANDER, 1985 BCSCF:

Marriage ceremony in BC between BC domiciliary and someone probably domicile in India (Mander) 7 days after ceremony, Mander left matrimonial home and disappeared Sangha asked for declaration of nullity or the marriage due to Mander’s impotence

I: What choice of law rule for essential validity of marriage should be applied wrt issue of impotence?R:

Apply dual domicile rule to determine essential validity of the marriage for question of impotence

A:

Presumption that unproven foreign law is the same as that of the forum – therefore law of B.C. applies for assessing capacity of both parties

Under B.C. law, impotence renders marriage essentially invalid and declaration of nullity given

CANADA V. NARWAL, 1990 FCAF: Marriage between Indian citizens in England

Wife had permanent residence status in Canada, husband did not After marriage, wife went back to Canada and husband wet to India - Question of essential validity of marriage on basis of affinity (wife had married her brother in law)

I: What choice of law rule for essential validity of marriage should be applied wrt issue of affinity?R:

Apply intended matrimonial home rule to determine essential validity of marriage for question of affinity Matrimonial home does not have to have been actually established (can just be intended) – the spirit

of the intended matrimonial home

47

Page 48: cans.allardlss.comcans.allardlss.com/.../cans/Edinger_76_Fall_2017_Jasmin…  · Web viewkConflict of Laws 2017. Some General Considerations. Keep in mind that while activities being

A:

Under dual domicile test – marriage in England is not essentially valid o Permitted under Canadian law (wife can marry brother in law under Canadian law) (wife has

capacity) o Not permitted under Indian law (husband does not have capacity)

Under intended matrimonial home test – marriage in England is essentially valid o Canada is going to be their intended matrimonial home, therefore apply Canadian law o Permitted under Canadian law therefore marriage is essentially valid o Fact that couple had not yet established a home here is not due to lack of interest or effort on

their part (due to inability to convince Canadian authorities) – spirit of the intended matrimonial home is met

SCHWEBEL V. UNGAR, 1965 SCR 148F: Two parties (U and W), domicile in Hungary, get married in Hungary

U and S divorced via Jewish divorce called a gett while in refugee camp in Italy (U’s domicile at time was Hungary)

Both parties form domicile of choice in Israel U moves to Canada, marries S (U’s domicile at this time is Israel); marriage breaks down S argues that their marriage was never essential valid because U was still married to W

o Israeli law (where U was domiciled at time of second marriage) considered divorce to be valid (and therefore U would have capacity to enter into second marriage)

o Italian law (where divorce was obtained) did not consider divorce valido Hungary law (where U was domiciled at time of divorce) did not consider the divorce valid o Canadian law had narrow rule for recognition of divorce: only recognize divorce obtained in the

domicile (divorce obtained in Hungary) and divorce recognized by the domicile (divorce recognized by Hungary)

Validity of second marriage depends on status of first marriage – was U validly divorced?I: How should the capacity of a party, who was previously married and purportedly divorced, to enter a

marriage be assessed: by the forum’s law for recognition for divorce or by the recognition of divorce law of the place where the party was domicile at the time of the subsequent marriage?

R: Determine whether a party had capacity to enter subsequent marriage (and therefore whether the subsequent marriage is essentially valid) based on the party’s status under the law of her domicile at the time of the subsequent marriage

A: At time of second marriage, U was domicile in Israel – Israeli law recognized the dissolution of the marriage

Therefore, U had capacity to marry S according to the law of the country where she was then domiciled (Israel)

C: The importance of this case is that the SCC had a choice to apply its own law or the law of U’s domicile and chose to apply the law of U’s domicile over its own law

- If apply own rules all the time, will get internal consistency; if apply the lex causae rules, won’t necessarily get internal consistency (may decide similar cases differently)

VERVAEKE V. SMITH, 1982 HLF: S and H get married; H moves to US and divorces S in Nevada

10 years later, S marries V (V was working as prostitute, was about to get deported so S agreed to marry her for money and a plane ticket); never live together

E, owner of valuable immovable property in London, and V get married; E dies on wedding night Fight between V and E’s brothers over who gets E’s immovable property (E died intestate)

o Depends on whether the wife was really a wife V goes to English, Belgian, and Italian courts for declaration that her marriage to S is invalid

o Argues that she didn’t understand the true nature of the ceremony o Or, even if she did understand, she didn’t really consent (this was sham marriage)

R: A primary question (ex: who gets property from deceased) can depend entirely on incidental questions of the validity of marriage(s)!

A: [English court applies own recognition rules to determine that US divorce is valid (S free to marry V)] English lower court held marriage between V and S was valid and never dissolved

o Therefore, V was not free to marry to E

48

Page 49: cans.allardlss.comcans.allardlss.com/.../cans/Edinger_76_Fall_2017_Jasmin…  · Web viewkConflict of Laws 2017. Some General Considerations. Keep in mind that while activities being

o Rationale: public policy in favour of upholding institution of marriage Belgian and Italian courts hold that she is married to E

o Rationale: public policy to not uphold sham marriages HL decided marriage between V and S was valid (res judicata and public policy) - Note: property is immovable property in question, so no choice but to apply English law to

determine who gets it (once determine the marriage validity issue)

TORTS

Note: remember that Moran locates a tort only for jurisdiction purposes (not for choice of law purposes)

Pre-Tolofson, courts applied the double-barrelled choice of law rule for torts (apply the forum law): the alleged tort has to be actionable in the forum and not justifiable by the law of the place where the tort occurred

o Tolofson changed this

TOLOFSON V. JENSEN, 1994 SCR 1022F: Car accident in SK between BC driver and SK driver; plaintiff was passenger of BC driver

Plaintiff brings action for damages in tort in BC courts Under SK law, 12 month limitation period (had passed) and higher level of negligence required

I: What is the choice of law rule for tort actions?R:

The general choice of law rule for torts is that a tort is governed by the law of the place where the tort occurred (lex loci delicti)

o But the general rule is not absolute—exception to general rule in the case of international torts in circumstances where the lex loci deliciti rule would work an injustice (where international = torts that have occurred outside of Canada); in this case, apply the law of the forum

L: Advantages for choice of law rule of lex loci delictio Certainty, predictability, ease of application, meets expectations of parties, discourages forum

shopping This rule is consistent with the Constitution and the real and substantial connection

A:

The proper choice of law to be applied is the law of Saskatchewan

SOMERS V. FOURNIER, 2000 ONCAF: Car accident in NY state between Ontario resident and NY state resident

Action started in ON; defendant wanted declaration that NY state law would apply Plaintiff wanted ON law to apply on basis that accident in NY was an international tort (better

outcome for plaintiff if ON law applied)I: Should the exception for int’l torts be applied to prevent an injustice, rather than the general choice

of law rule, for determination of which substantive law to apply in this tort action? How are various tort issues characterized (substantive or procedural)?

R:

Courts retain a discretion to apply the local law in international litigation of a tort where necessary to avoid injustice but this should be exercised only in exceptional and compelling circumstances

Characterization of issues related to torts (warning: this is just ONCA’s characterization, might vary)o Costs: procedural (apply forum law)o Availability of pre-judgment interest: substantive (apply law of place where tort occurred)o Quantification of damages: procedural (apply forum law)o Cap on damages: procedural (apply forum law)

A:

Do not have an appeal level case where court has been persuaded to make an exception to the lex loci deliciti choice of law rule for torts

o There is nothing exceptional about this case that makes it different from those cases Therefore, the lex loci delicti rule governs and NY state law will apply

49

Page 50: cans.allardlss.comcans.allardlss.com/.../cans/Edinger_76_Fall_2017_Jasmin…  · Web viewkConflict of Laws 2017. Some General Considerations. Keep in mind that while activities being

EDITIONS ECOSOCIETE V. BANRO, 2012 SCC 18 (NO NOTES ON IT; IN TEXT AT PAGE 681)F: Banro, Ontario company, sued EE for alleged defamation contained in a book

93/5000 books distributed in Ontario and copies available in Ontario public libraries Litigation in Ontario

I: What is the choice of law rule for the tort of defamation? R: Re Jurisdiction

The commission of a tort in forum is recognized presumptive connecting factor that prima facie entitled forum to assume jurisdiction. Unless D’s can show that only minor element of the tort of defamation occurred in forum, fail to displace presumption of jurisdiction.

Re Forum Non Conveniens Onus on Defendant to demonstrate that more appropriate forum based on R&S connecting

factors. L: The tort of defamation is crystallized upon publication of the libellous material. In Canada, publication

occurs when libellous material is read by a third party. In the case of libellous material printed in a book that is circulated in a library, it is possible to draw an inference of publication

Wrt forum non conveniens Ds argued a) cost of proceeding in Ontario, b) existence of parallel proceeding in Quebec, c) that QB law governs; d) and that relative juridical advantage and disadvantage of the parties all favoured Ontario court staying action via exercise of discrection (finding itself forum non conveniens). a) essentially argument of proportionality which rests on lack of evidence of harm, which is not concern at preliminary stage of proceeding. Harm typically presumed in defamation cases unless sufficient evidence to contrary produced. b) Court says they are not actually parallel proceedings, two defamation proceedings involving same book but different plaintiffs. No guarantee that if Ontario exercised discretion actions would be amalgamated in Quebec. These are separate claims that although arising from the same book, the claims are related to different allegedly defamatory statements. c) Choice of Law: “One factor that must be considered in forum non conveniens analysis is the law applicable to the tort”—QB basically arguing applicable law is where majority of publication was. Le Bel J. says no, applicable law is still lex loci delicti—but muses about case where there are publications in multiple jurisdictions (below). In this case, either way, applicable law is that of Ontario. d) Juridical advantage—on one hand Ds lose procedural advantage of particular QB statute, on the other hand, if stayed in Ontario, Ps might face argument that its claim is barred in QB on basis of the short 1 yr limitation period for defamation claims under art 2929 of CC of QB. Weighing, balance of fairness favours banro. Also, even though connection b/w more than one forum, given strength of connection b/w P and Ontario, not clear Ps forum shopping.

A:

Ontario has jurisdiction b/c lex loci delicti rule—tort occurred in Ontario. Even if musings were the appropriate approach (place of most harm) Ontario is palce where Banro suffered most harm. D’s fail to show on BoP that QB more appropriate forum than Ontario. They tried to argue based on cost of proceedings, alleged existence of parallel proceedings in QB, applicable law, and juridical advantage, but none of these arguments were accepted.

C: Le Bel Muses about a choice of law question which may impact decision to exercise discretion or not at forum non conveniens stage. While place of most substantial publication does not impact choice of law for defamation, he considers that it may be—instead of lex loci deliciti—that where defamatory material is published in more than one place, the applicable choice of law rule is the law of the place where the most substantial harm to P’s reputation was suffered.

Not what we studied this case for, but Banro says this re Forum Non Conveniens at para 41:

The application of forum non conveniens is an exercise of discretion reviewable in accordance with the principle of deference to discretionary decisions: an appeal court should intervene only if the motion judge erred in principle, misapprehended or failed to take account of material evidence, or reached an unreasonable decision (see Young v. Tyco International of Canada Ltd., at para. 27). I find that the motion judge made no such error. Roberts J. correctly exercised her discretion in maintaining Ontario’s jurisdiction over the plaintiff’s claim.

CONTRACTS

Remember: no renvoi in contracts

I. The Proper Law

50

Page 51: cans.allardlss.comcans.allardlss.com/.../cans/Edinger_76_Fall_2017_Jasmin…  · Web viewkConflict of Laws 2017. Some General Considerations. Keep in mind that while activities being

Primary contracts choice of law rule: contracts are governed by the proper law of the contract

Why: contracts generally defer to party autonomy - The proper law of the contract exists from the beginning (contract cannot exist in a vacuum) and does not

change - Possible for contract to have more than one proper law- Proper law is the law the parties intended, either expressly or implicitly, to govern the K, and in

the absence of any such intention it is the law with which the transaction has its closest and most real connection.

How courts determine the proper law of the contract – options

- Via an express choice of law clause included in the contract (Vita Foods)- If no express choice of law clause in contract

o Determine the implied choice of law (Colmenares), oro Determine the objective proper choice of law (strongest connection to transaction)o Note: hard to tell which one of these two the court is doing

When you are thinking about a K from a conflicts perspective, at least 4 possible connections to particular countries which may or may not coincide: 1) forum where dispute litigated, 2) place of formation of K (may be different from forum), 3) Place of substantial performance of K, 4) Proper law of the K (may not be any of the above if parties chose something else)

Note: the proper law of the K must be determined as of the time/date when the contract was formed. But proper law not static, it will be proper law as it stands at time of dispute.

VITA FOODS V. UNUS SHIPPING, 1939 PC (APPEAL FROM CANADA)FACTS Unus carrying goods on ship from Nfld (part of England at time) to New York; contract

made in Nfld Ship damaged in storm off of NS, litigation proceeds in NS (appropriate forum) Contract contained 2 relevant clauses: (1) D not liable for damage arising from

negligence of master and (2) contract governed by English law P argued that D failed to comply with Nfld statute wrt contents of contract (Nfld law

said all bills of lading had to incorporate Hague Rules), that K therefor void and that the Ds could not rely on void contract to exempted them from negligence and were therefore liable for the damage

In NS courts, proceeded on basis of lex loci contractus – law that governs the contract is the law of place where the contract was made (Nfld); no one argued that English law should apply

ISSUE 1) How is proper law of K determined? Is it law of place where K made, in this case Nfld? 2) Was failure to obey Nfld Act illegal under law of nfld? 3) Does failure to obey law of nfld render K void?

RATIO The proper law of the contract is the law that the parties intended to apply to the contacto An express choice of law will apply as long as the intention is bona fide and

not contrary to public policy LAW The proper law of the contract is the law which the parties intended to apply and this

intention is objectively determined(and you need to determine the proper law of K to determine if any particular act creates illegality problem)o Where there is an express statement by parties to select the law of the contract,

that will apply (objective evidence of intent) provided that the intention is bona fide and legal and no reason for avoiding the choice on the ground of public policy

o If intention not expressed, will be presumed from the terms of contract and surrounding circumstances

Proper law of the contract does not require any connection between the parties and the law selected to govern the contract to be bona fide.

Parties can also incorporate other law (ex: a statute of a legal system that is not the proper legal system) and this does not displace the proper law of the contract. It may also be the proper law incorporates provisions of the law of another country, but that does not change proper law of K.

Also, where K illegal in one jurisdiction, forum not bound to treat K as illegal if would not be illegal by proper law of K. However, where forum and law purporting to impact legality of K are the same, then position might be different: “A court in Nfld would be bound to apply the law enacted by its own legislature..[gives USA example where…] such clause, it was held, was against pp and void by the law of the US, which was not

51

Page 52: cans.allardlss.comcans.allardlss.com/.../cans/Edinger_76_Fall_2017_Jasmin…  · Web viewkConflict of Laws 2017. Some General Considerations. Keep in mind that while activities being

only the law of the forum but was also held to be proper law of K. (p 699)o TAKE AWAY—what is proper law of K and what is law of forum, may be that

forum can ignore illegality if not illegal by reason of forum’s own law or law of K, but through some other cxn like place of formation or performance).

When considering illegality you need to determine whether law is directory or obligatory but trying to get at the “real intention of the legislature by carefully attending to the whole scope of the statute” (per Turner). Even if illegal, unless forum probably entitled to ignore obligatory statute that is not law of your forum or proper law of the K: “…it does not follow that any other could could properly act in the same way. If it has before it a K good by its own law or by the proper law of the K, it will in proper cases give effect to the K and ignore the foreign law” (p. 701).

APPLICATION Express choice of law clause selecting English law as the proper law of the contract Wrt breach of the Nfld statute: should NS/English court take notice of Nfld statute and

the breach?o If statute is merely directory, no need to comply with ito If statute is obligatory, NS/English court still not bound to take notice of it (can

ignore illegality)o In this case, the statute is merely directory (even in Nfld, contract would not be

void)

RICHARDSON INTERNATIONAL LTD. V. MYS CHIKHACHEVA, 2000 FCAFACTS R is Washington state business, supplied necessaries for Russian ship M

R not paid, causes M vessel to be arrested in BC (in rem action suing the ship) Existence of the lien against the ship depended on proper law of the K b/w R and the

company that owned the ship (no express choice of law clause in the contract)—it mattered b/c Canadian law did not allow for maritime lein to suppliers but American law did (Russian law did not come into play b/c not proven at trial).

ISSUE How to determine the proper law of the contact when there is no express choice of law clause? (we’re looking for implied choice or objective law of K, not clear which)

RATIO If there is no express choice of law, one option the court has is to determine whether the parties, on the objective interpretation of their contract, actually did reach an implicit agreement on what the proper law should be

LAW Weigh all the relevant factorso Presence of an arbitration clause – if parties agree that arbitration should

take place in particular legal unit, court will usually conclude that parties have impliedly chosen the law of the legal unit of arbitration as the proper law of the K. Court says “while not determinative” and arbitration clause is “highly persuasive” of parties implied intention re proper law of K (para 34).

o Presence of jurisdiction selecting clause – same as above o Other factors: legal terminology in contract, form of documents in the txn,

currency in which payment is to be made, use of a particular language, connection with a preceding txn, nature and location of the subject matter of the contract, residence (but rarely nationality) of the parties, head office of corporate party, fact that one of the parties is gov’t

o Not determinative but highly persuasive General rule that party may not give evidence as to his or her subjective intention at

the time the contract was made APPLICATION Factors

o Arbitration clause in contract which chose to arbitrate in Seattle – this is indicative of parties implied’ intentions to have American law apply

o Also, legal terminology and form favour American law (drafted by US lawyers, currency expressed in US dollars), cxn to proceeding txns also relevant b/c marketing K incorporates by addendum reference promissory note and mortgage.

Therefore, American law is the proper law of the contract

This case sets out the proper process for determining proper law of K (para 28): 1) Court determines if there is an express choice of law by the parties. If there is none, the 2) the court must determine whether the proper law can be inferred from the terms of the K and the surrounding circumstances…an exercise that requires the Court to determine the system of law that has the closest and most real connection to the K.

52

Page 53: cans.allardlss.comcans.allardlss.com/.../cans/Edinger_76_Fall_2017_Jasmin…  · Web viewkConflict of Laws 2017. Some General Considerations. Keep in mind that while activities being

Court also talks about the parties intent that their “relationship be governed by a complex series of interrelated components, and not discrete, stand alon Ks…” (para 30). This might be useful where a package of Ks but only one has the thing you are looking to apply and you want to try and connect it to others somehow.

What if no express or implied proper law of K? Then you have to objectively determine the proper law of the K. Relevant factors include:

1) Place K was made (usually not that weighty)2) Parties of the K—where do they carry on business? where are they domiciled? What countries are they citizens of? 3) Then look at the K itself: what is the language of the K, what terms are used? Are these terms that valid/utilized in

one system but not the other? What is the currency of account? What is the validity/effectiveness of respective clauses under different systems? Is there a forum or arbitration selection clause (these are generally considered pointer towards correct CoL decision)?

Even once you decide proper law of the K you have some options—for example, is this an issue that deals with formation of K, capacity of the parties, illegality? Might be able to characterize rules of forum as procedural and have them applied even though proper law of K is not that of forum.

IMPERIAL LIFE ASSURANCE CO. OF CANADA V. COLMENARES, 1967 SCCFACTS C purchased life insurance policy while in Cuba

K written in Spanish, but standard form K used in Ontario (head office located there) C wanted to cash in on life insurance policy (no longer in Cuba) If proper law of K was Cuban law, could not cash in (due to foreign exchange controls) Court decides that the K was made in Ontario

ISSUE How to determine the proper law of the contract objectively? RATIO The proper law of the contract is determined objectively by considering the contract as a

whole in light of all the circumstances that surround it and applying the law with which it has the closest and most substantial connection

- You can also cite this case for proposition that the proper law of the K is determined as of the date the k was made

LAW Factors to consider1. Domicile and residence of the parties2. National character of a corporation and the place where its principal place of business

is situated or where head office is located 3. Place where the contract is made and where it is to be performed 4. Style in which the contract is drafted (ex: is language appropriate to one legal system

but not another)5. The fact that certain stipulation is valid under one law but void under another 6. Economic connection of contract with some other txn 7. Nature of the subject matter 8. Any other fact which serves to localize the contract

APPLICATION Factors o Location of the head office is significant (actual decision about the contract was

made there, not in Cuba)o Also, applications and polices prepared in Ontario, standard form of Ontario

Therefore, proper law of the contract is Ontario

AMIN RASHEED SHIPPING CORP. V. KUWAIT INSURANCE, 1984 HLFACTS P was Liberian company carrying on business in Dubai, D was Kuwaiti insurance

company P’s ship taken over by pirates, want to make claim on insurance K for total loss K written in English, in form in accordance with English act, money out account was

English But k made in Kuwait and payable in Kuwait; but Kuwait did not have body of marine

insurance law at time contract was entered into No express choice of law clause in the contract

ISSUE What is the proper law of the contract?

53

Page 54: cans.allardlss.comcans.allardlss.com/.../cans/Edinger_76_Fall_2017_Jasmin…  · Web viewkConflict of Laws 2017. Some General Considerations. Keep in mind that while activities being

RATIO If there is no express choice of law in the contract, the proper law of the contract is the system of law by reference to which the contract was made or that with which the transaction has its closest and most real connection

- The “or” is disjunctive (mutually exclusive)- Look at the contract or at what is being done under the contract

APPLICATION - Diplock: the parties actually intended English law to apply - Wilberforce: consider the relevant factors to determine the proper law objectively – weigh in favour or English law as the proper law of the contract b/c weight given to form and language of K pointing to law of England.

COMMENTARY - Analysis is easier if use the reference to the disjunctive “or”- Factors connected with the contract: 4 corners of the contract, the language contract is written in, the terms used in the contract (ex: term used only in civil law jurisdiction or valid only in some legal jurisdictions), money of account, existence of arbitration or jurisdiction selecting clause- Factors connected with what is to be done under the contract: place in which the contract is made, place of performance, residence of the parties, nationality of the parties

RE POPE AND TALBOT, 2009 BCSCCase where different parts of K found to be gov’d by different proper lawFACTS P&T go into receivership, claimed protection under the CCAA

Former employees of P&T make wage claim; P&T want to make claim on D&O insurance policy to cover the wage claim

To determine whether insurance policy will cover wage claim, had to determine whether contract was governed by BC law or Oregon law or by more than one proper law

ISSUE Can a contract be governed by more than one proper law?RATIO Different contractual issues in a contract can be governed by more than one proper law

(but is rare)o Must be an express or implied agreement of this

APPLICATION Clear intention that parties intended for more than one proper law to apply to different parts of the contracto Parties had lots of opportunities to choose one proper law but chose not to choose,

deliberatelygu no CoL clauseo Language in policy contemplates or allows for different parts of the policy to be

governed by different legal regimes Proper law of contract wrt insurer’s obligations for wage claims is B.C. law

In a case, the D is looking around for defences (ex: D wants the court to find the contract void)

- Contract may have connections with many different legal systems – D can try to point to one of those o Legislatures regulate contracts so likely a good selection of statutes in the various connected legal

system that D can choose (ex: some breach (formation, formality, illegality) that makes the contract void under that legal system

II. Formation

- Situation: P is arguing that there is a contract and is suing on it; D is arguing that there never was a contract - Question: did the contract come into existence?

Two possible choice of law rules for formation of a contract – have a choice!

- (1) The formation of a contract is governed by the recognized proper law (Albeko, 1961)o The Parouth (1982, Eng CA)

P sue Ds on contract, D argues no contract; question of whether Eng CA could take jurisdiction Eng CA applies rule that formation of contract is governed by putative proper law

One document included arbitration clause selecting London Therefore, if there was to be a contract, the putative proper law is English law

- (2) The formation of a contract is governed by the law of the forum (Mackender, 1967 Eng CA)

54

Page 55: cans.allardlss.comcans.allardlss.com/.../cans/Edinger_76_Fall_2017_Jasmin…  · Web viewkConflict of Laws 2017. Some General Considerations. Keep in mind that while activities being

MACKENDER V. FELDIA AG, 1967 ENG CAFACTS Insurance policy agreement between Lloyds and diamond merchants

Contract had choice of law and exclusive jurisdiction clauses selecting Belgium Diamond merchant suffers loss and makes claim on insurance policy Despite choice of law and jurisdiction clauses, Lloyds wanted the action in England:

argues that merchants did not disclose that they were involved in diamond smuggling, therefore contract is void for non-disclosure (jurisdiction clause does not apply) and can litigate in England

Diamond merchants want the dispute to be decided in Belgium In order to decide if English action should be continued, had to decide whether contract

was formedISSUE What law should be applied to determine whether a contract has been formed or has

been rendered voidable?RATIO To determine whether a contract has been formed, apply the law of the forumAPPLICATION Contract was formed according to English law; jurisdiction selecting clause stands therefore

settle dispute in Belgium

“Where an agreement is wholly unenforceable b/c it is contrary to English law, it may, if the proper law of the agmt is itself English law, accurately be said to be void as a K…a k is a species of agmnt which gives rise to legally enforceable rights and duties; and an agmt which is contrary to English law, if that is also its proper law, gives rise to none. But if an agmt, though contrary to English law, is not illegal by its proper law, it cannot propyl said to be void and thus not a K at all. It does give rise to rights and duties which are legally enforceable elsewhere than in England. It is a K, but one which is unenforceable in English Courts…this is to be contrasted with an agmnt which under its foreign proper law is illegal and incapable fo giving rise to legally enforceable rights and liabilities under that law. Since the foreign prper law must be looked to for the legal effects of the agmnt, such an agmnt may properly be said to be void.u” (743).

III. Formalities

- Are there any statutes that say certain procedures have to be followed in order for contract to be formally valid? EE says not generally big issue but could make or break a particular case. Possibly wrt online Ks and legislation surrounding them, or protective statutes, e.g. Ks w/ Doctors.

GREENSHIELDS INC. V. JOHNSON, 1981 AB QBFACTS - At time, AB had Guarantees Acknowledgment Act which required guarantor to sign a

guarantee in front of a notary - In this case, contract of guarantee was entered into in AB but act not complied with (no notary used)- Contract had choice of law clause selecting the law of Ontario

ISSUE How is the formal validity of a contract determined?RATIO To determine whether a contract is formally valid, there are alternate choice of law rules

(can use whichever rule produces a valid contract)- (1) Law of the place where the contract was entered into (lex loci contractus), or - (2) The proper law of the contract

APPLICATION - No reason to invalidate Ontario as the choice of law for the proper law of the contract (Vita Foods)- Under (1) law of the place where the contract was entered (AB), the contract is invalid for failing to comply with the Act- But under (2) the proper law of the contract (ON), the contract is valid because ON does not have a similar statutory requirement for guarantees

COMMENTARY ABCA characterize the issue differently: issue of substance or procedure- Guarantees Acknowledgment Act is substantive – forum (AB) cannot apply its own

law to contract that has selected different substantive law (ON) so contract is valid - Note: if CA had characterized statute as procedural, forum would apply the

statutory requirement to the contract since it was made in AB- **Remember that you have characterization options too!

IV. Illegality: Rules of Mandatory Application

- Situation: P wants to sue D on the contract; D can plea that contract is illegal and therefore cannot be enforced

55

Page 56: cans.allardlss.comcans.allardlss.com/.../cans/Edinger_76_Fall_2017_Jasmin…  · Web viewkConflict of Laws 2017. Some General Considerations. Keep in mind that while activities being

o D will look around at the different jurisdictions that contract is connected to and try to find a jurisdiction with a statute that will help D plea illegality

- Rules of mandatory application = statutes that regulate contracts o Question: which statutes are rendered applicable by the choice of law rules?

4 different connections between contracts and legal jurisdictions (Vita Foods)

- (1) Where the contract was made (lex loci contractus)o If only connection is that contract happened to be made in that jurisdiction, Vita Foods says ignore it

Never applies unless the lex loci contractus happens to coincide with the forum (Avenue)- (2) Place where the contract is to be performed

o Always applies, subject to interpretation- (3) Proper law of the contract

o Note: no cases dealing with illegality due to proper law of the contract because no doubt that statute of the proper law of the contract applies to the contract

o Always applies, subject to interpretation - (4) Forum (of litigation)

o If forum law is obligatory, forum cannot ignore and must apply. - Note: these may overlap some, totally, or not at all

Vita Foods v. Unus Shipping, 1939 PC

- Forum (English conflicts rules) can disregard illegality due to breach of statute that is part of law of place where the contract was made

o If the contract is good by forum’s own law or by the proper law of the contract, forum can give effect to the contract and ignore the foreign law

- If, however, forum is the same place as the place where the contract was made, then forum court cannot disregard illegality

o If suit was brought on contract in Nfld court and court held the contract was illegal for failing to comply with some Nfld statute, the court would refuse to give effect to contract on basis that court is bound to obey the laws of its own legislature or its own CL

AVENUE PROPERTIES LTD. V. FIRST CITY DEVELOPMENT CORP, 1986 BCC connection = forumFACTS Deal between Avenue (BC company) and First City (ON company) for Avenue to

purchase units in Ontario development (immovable property in ON); in K, parties agreed that proper law of contract was Ontario (express choice of law clause) and that parties would attorn to jurisdiction of Ontario (not the same as exclusive jurisdiction clause)

Avenue backs out; First City sues Avenue in Ontario for specific performance Later, Avenue applies for declaration in BC court that K is not enforceable because of

BC statute (BC Real Estate Act) which requires prospectuses be issued to BC purchaser regardless of whether property is in BC and no prospectus was received

Jurisdiction case (is BC the appropriate forum)ISSUE How does a law of mandatory application that would operate to make a contract

unenforceable factor into a court’s consideration of forum non-conveniens?RATIO If a local law is a law of mandatory application that would operate to give a party a juridical

advantage, that weighs in favour of the local court retaining jurisdictionLAW Court can apply the law of its own jurisdiction in substitution for proper law of contract

when:o [Applying the proper law of the K would be contrary to forum public policy]o The local law is procedural o The local law, even though substantive, is of such a nature that it should be

applied (law of mandatory application) – the court must do this when local legislation specifically states that certain procedures will apply, notwithstanding that the proper law of the K may indicate otherwise

APPLICATION In Real Estate Act, legislature has specifically said that persons soliciting the sale of land in BC, whether the land is inside or outside the province, must comply with the Act’s prospectus requirement

o There is at least a reasonable possibility that a B.C. court would accept the choice of law argument that the legislature intended this provision to apply, notwithstanding that land and proper law of the K were foreign

o Ontario court would likely not apply the provision (would characterize it as

56

Page 57: cans.allardlss.comcans.allardlss.com/.../cans/Edinger_76_Fall_2017_Jasmin…  · Web viewkConflict of Laws 2017. Some General Considerations. Keep in mind that while activities being

procedural or if it characterized it as substantive, would not apply it on basis that parties selected ON law as proper law of the K)

Therefore, Avenue established that there is a fair possibility that it will gain a legitimate juridical advantage by prosecuting its action in BC rather than ON; do not stay BC action

COMMENTARY This is example of hypothetical from Vita Foods

GILLESPIE MANAGEMENT CORP. V. TERRACE PROPERTIES, 1989 BCCA connection = place where contract was to be performedFACTS T is Washington State partnership that manages properties in Washington State; G is

BC company T not licenced in Washington to manage properties (in breach of statute that requires

licencing) K between T and G made in BC, court found that it was governed by BC law Litigation in BC

ISSUE Should the BC court consider/apply the law of Washington (the statute that requires licencing)?

RATIO Two possible approaches: Doctrine of illegality is founded on forum public policy – have public policy of not

enforcing unlawful bargains,o Give effect to foreign legislation which is of the same order as domestic

legislation Illegality by the law of the place of performance of the contract is relevant and to be

consideredAPPLICATION By approach 1

o BC has act that makes it unlawful to act as agent for collecting rents unless have a licence

o As matter of forum public policy, give effect to foreign public policy analogous to our own

By approach 2o Actions were illegal by WA statute, place of performance, htherefore do not

enforce the contract COMMENTARY Next step: after the court decides that one of the connections is relevant, it is not the end –

the court still has to decide what the impact on the K is (engage in statutory interpretation – read statute, get expert evidence, and decide whether or not the statute, when applied to the K, would make the K void)

P 767: “The Anglo-Canadian private international law of Ks has so far developed no rules to give effect to the regulatory interests of (1) any countries other than the country of the proper law; (2) the forum (whose law applies if the local legislature directs the court to apply it to the contract); and (3) the law of the place of performance (which is applied if the issue relates to the “mode of performance” or to illegality of an act of performance”).

UNJUST ENRICHMENT

Dicey Rule 230

- (1) The obligation to restore the benefit of an enrichment obtained at another’s expense is governed by the proper law of the obligation

- (2) The proper law of the obligation is determined as follows:o (a) If the obligation arises wrt a contract, the proper law is the law applicable to the contract;o (b) If the obligation arises wrt a transaction concerning an immovable, the proper law is the

law of the place where the immovable is located (lex situs); o (c) If the obligation arises in any other circumstances, the proper law is the law of the country in which

the enrichment occurs. - BUT, it is not clear from the rule which of these should apply if there is a contract and an immovable!

MINERA AQUILINE ARGENTINA SA V. IMA EXPLORATION INC. AND INVERSIONES MINERA

57

Page 58: cans.allardlss.comcans.allardlss.com/.../cans/Edinger_76_Fall_2017_Jasmin…  · Web viewkConflict of Laws 2017. Some General Considerations. Keep in mind that while activities being

ARGENTINA SA, 2006 BSCS; AFFD 2007 BCCAFACTS Company selling Mine Site (BLEG B) and IMA interested but wants more info. Company

says okay come look, but you have to sign confidentiality agmt. IMA finds out about more valuable mine while visiting BLEG B. Declines to buy, then has subsidiary in Argentina buy BLEG A. Minera buys BLEG B and then says hey! You breached K with Company and we should have been the ones to benefit from that info about BLEG A, not you jerks.

Minera alleged that IMA used confidential information about land in Argentina and used it to purchase site nearby the site that was being offered for sale

K was probably governed by law of Colorado but no expert evidence offered as to K law of Colorado therefore BC law applied (if no expert evidence on lex causae law, then assume it is the same as forum law and apply forum law)

Court found that confidential information used here was covered by the confidentiality agreement in the K (resolved merits of case without having to consider conflicts issues) but went on to discuss the conflicts issues in case that issue not covered by the K

ISSUE How to determine the proper law of the obligation? Are Dicey’s rules intended to be hierarchical?

RATIO In choosing the appropriate law to govern a claim of unjust enrichment when there is a contractual relationship and a transaction involving immovable property, apply the law that has the closest and most real connection to the obligation

o Resolve any apparent clash between the first two sub-rules by taking a principled rather than categorical approach to the choice of law issue

LAW When obligation arises in connection with both a pre-existing contractual relationship and a transaction involving foreign land, examine all the factors that could be relevant to the strength of the connection between the obligation and the competing legal systems

o Give factors weight according to a reasonable view of the evidence and their relative importance to the issues at stake

Non-exhaustive list of factors to determine which set of laws has the closest and most substantial connection to the obligation

o Where the txn underlying the obligation occurred or was intended to occur o Where the txn underlying the obligation was or was intended to be carried out o Where the parties are resident o Where the parties carry on businesso What the expectations of the parties were wrt governing law at the time the

obligation aroseo Whether the application of a particular law would cause an injustice to either of

the parties APPLICATION Factors

o Enrichment occurred in Argentina, both parties carried on business in Argentina and where data was intended to be used

o But neither party was Argentine company and none of principals involved were Argentinian (Canadians or Americans who lacked understanding of Argentine law wrt confidential information) so unlikely that parties would have chosen or expected Argentine law to govern their actions and relationship; were familiar with Canadian and American law on this issue

o Therefore, BC law has closest and most real connection to obligation between these parties

COMMENTARY This case is possible as an exception to the Mocambique rule (exception if there is an equitable claim or contract claim; in this case, is an equitable claim of unjust enrichment/breach of confidentiality)

- There has not been another conflicts choice of law case dealing with unjust enrichment since this case, so not clear if what is made into a single choice of law rule for this situation will catch on

PROPERTY

For any property question, first thing you must do is to classify the property as immovable or movable!! (HOGG)

Categories of property issues

58

Page 59: cans.allardlss.comcans.allardlss.com/.../cans/Edinger_76_Fall_2017_Jasmin…  · Web viewkConflict of Laws 2017. Some General Considerations. Keep in mind that while activities being

- Inter vivos transfers of title to property o If classified as property issue: Cammwello If classified as contract issue: proper law of the K

- Post-mortem transfers of title to property o Administration of estates o Succession

Intestate Movable Immovable

Testamentary succession Movable

o Formal validityo Essential validity

Immovable o Formal validity o Essential validity

- Affect of marriage and marriage break down on property - Trusts

o Inter vivos trusts o Testamentary trusts

I. Movables: Inter Vivos Transfers

CAMMWELL V. SEWELL, 1860 COURT OF EXCHEQUERFACTS Lumber being shipped from Russia to England but doesn’t make it (ship wreck in

Norway) Consignee in England makes claim on insurance and gets 100% back Insurance company wants to be paid back because lumber was sold to someone else Norwegian court confirmed that the sale of the lumbar was valid

ISSUE What is the choice of law rule for the inter vivos transfer of movable property?RATIO Choice of law rule for inter vivos transfers of movable property is the law of the place

where the movable property is located at the time of the transfer APPLICATION Norwegian law conclusively determined title to the lumber

WINKWORTH V. CHRISTIE MANSON AND WOODS LTD., 1980 CHRATIO In dicta, this case suggests that English court might use renvoi in connection with lex situs

at the time of the transferCOMMENTARY In 2007 case, Queen’s Bench rejected introducing renvoi into transfer of title to movable

property; so likely don’t need to worry about it

II. Succession: Wills and Trusts

Wills

- Wills, Estates, and Succession Act, SBC 2009 Bill 4, ss. 79-83 and Schedule 2 o Codifies the CL but incompletely o Provides many independent options for finding a will to be formally valid o But does not include choice of law rule for determining essential validity of a will

Have to go to the CL Therefore, can still use renvoi for essential validity of a will

- Common law rules for essential validity of a willo Cannot choose which law you want to govern your will o Two different choice of law rules, depending on if property is movable or immovable

Movable property: essential validity of will is governed by domicile of the testator at the date of death (Aguilian)

Immovable property: essential validity of will is governed by the lex situs This means that will might be half valid and half invalid!

o Can, in theory, use renvoi

59

Page 60: cans.allardlss.comcans.allardlss.com/.../cans/Edinger_76_Fall_2017_Jasmin…  · Web viewkConflict of Laws 2017. Some General Considerations. Keep in mind that while activities being

Trusts

- CL concept o Had Hague Convention on law applicable to trusts so that civil law jurisdictions will recognize trusts

created in CL jurisdictions (ex: if property subject to trust is in civil law jurisdiction)- Whatever document creates the trust (the will, trust document, etc) must be independently valid

o Ex: if trust created as part of a will, the will must be formally and essentially valid before can even consider the will

- In B.C., have two statutes that govern trusts

o International Trusts Acts, RSBC 1996 c. 237 The trust settlor can select the law to govern the trust and the administration of the trust Provides criteria for court to consider in the event that the settlor fails to choose the law to

govern the trust

60